Logical Reasoning Questions

¡Supera tus tareas y exámenes ahora con Quizwiz!

James: Community colleges, by their very nature, work to meet the educational needs of the communities they are in. The same is not true of universities, whose primary goals differ from those of community colleges. Margaret: A primary goal of any university is to serve the needs of the community where it is located. The main reason people have for attending a university is the same as that for attending a community college: preparing oneself for a career. James's and Margaret's statements provide the most support for the claim that they disagree over the truth of which of the following? (practice test 3) A) A primary goal of any university is to serve the educational needs of its community. B) Most universities adequately serve the educational needs of the communities in which they are located. C) The main reason people have for attending a university is to prepare themselves for a career D) In a typical community, the primary educational need is to prepare community residents for careers. E) The main reason people have for attending a university is the same as the main reason people have for attending a community college.

A) A primary goal of any university is to serve the educational needs of its community. James claims that the primary goal of a university is NOT to serve the educational needs of its community, while Margaret claims otherwise. B: Neither comments concern whether the universities actually DO serve the needs of their community C: James doesn't have an opinion on this D: James doesn't express an opinion about the specific educational needs of a given community E: James doesn't say anything about why people attend university or community college

Field studies, which have long been a staple of anthropological research, involve the researcher living within the community being studied. However, the usefulness of field studies tends to be overrated by anthropologists. Although most anthropologists do realize that living within the community one is studying affects that community, they generally underestimate the extent of such effects. Which one of the following most accurately expresses the conclusion drawn in the argument? (practice test 3) A) Anthropologists tend to overestimate the value of field studies. B) In a field study, the researcher lives within the community being studied. C) Field studies have been a central feature of anthropological research for a long time D) Most anthropologists know that when they live within a community being studied, the community is affected at least somewhat. E) Most anthropologists underestimate how much of an effect the researcher's presence has on a community being studied

A) Anthropologists tend to overestimate the value of field studies. Keyword: However

A recent poll of a large number of households found that 47 percent of those with a cat had at least one person with a university degree, while 38 percent of households with a dog had at least one person with a university degree. Clearly, people who hold university degrees are more likely to live in a household with a cat than one with a dog. The reasoning in the argument is flawed in that the argument (practice test 11) A) ignores the possibility that a significant number of households might have both a cat and a dog B) takes for granted that there are not significantly more households with a dog than ones with a cat C) fails to consider how many of the households have at least one person without a university degree D) fails to consider to what extent people with university degrees participate in decisions about whether their households have a cat or a dog E) ignores the possibility that two things can be correlated without being causally connected

B) takes for granted that there are not significantly more households with a dog than ones with a cat

Any good garden compost may appropriately be used for soil drainage and fertility. The best compost is 40 to 60 percent organic matter and is dark brown in color. However, compost that emits a strong ammonia smell should not be used for drainage and fertility, for that smell means that the organic matter has not sufficiently decomposed. Which one of the following is most strongly supported by the information above? (practice test 10) A) Compost that is 80 percent organic matter has probably not decomposed sufficiently. B) If compost is less than 40 percent organic matter and is not dark brown in color, then it will make soil less fertile and will worsen soil drainage. C) If compost is 50 percent organic matter and that organic matter is sufficiently decomposed, then the compost is good D) In the best garden compost, the organic matter is completely decomposed E) Compost that is dark brown in color and emits a strong ammonia smell is not good garden compost

E) Compost that is dark brown in color and emits a strong ammonia smell is not good garden compost Color is irrelevant, the rest is supported by the passage whereas the other choices are not

The caffeine in coffee stimulates the production of irritating acid in the stomach. But darker roasts of coffee, produced by roasting the coffee beans longer, contain more N-methylpyridinium (NMP) than lighter roasts, and NMP tends to suppress production of acid in the stomach. Therefore if you drink caffeinated coffee, darker roasts will irritate your stomach less than lighter roasts. The answer to which one of the following questions most helps in evaluating the argument? (practice test 8) A) Does extending the roasting time of coffee beans increase the amount of caffeine present in the brewed coffee? B) Does a reduction in acid production in the stomach have an adverse effect on stomach function? C) Would coffee drinkers who drink caffeinated coffee increase their coffee consumption if the coffee they drank contained less caffeine? D) Do some coffee drinkers who switch from lighter to darker roasts of coffee increase their daily coffee consumption? E) Do lighter roasts of coffee have any important health benefits that darker roasts of coffee lack?

A) Does extending the roasting time of coffee beans increase the amount of caffeine present in the brewed coffee? This helps; if the answer is yes, the argument is weakened because darker roasts would provide more caffeine, stimulating the production of irritating acid in the stomach B: We aren't concerned with stomach FUNCTION, but rather IRRITATING the stomach

A tax preparation company automatically adds the following disclaimer to every e-mail message sent to its clients: "Any tax advice in this e-mail should not be construed as advocating any violation of the provisions of the tax code." The only purpose this disclaimer could serve is to provide legal protection for the company. But if the e-mail elsewhere suggests that the client do something illegal, then the disclaimer offers no legal protection. So the disclaimer serves no purpose. The argument's conclusion can be properly drawn if which of the following is assumed? (Practice Test 6) A)If the email doesn't elsewhere suggest that the client do anything illegal, then the company does not need legal protection. B) If email messages sent by the tax preparation company do elsewhere suggest that the recipient do something illegal, then the company could be subject to substantial penalties. C) A disclaimer that is included in every email message sent by a company will tend to be ignored by recipients who have already received many emails from that company. D) At least some of the recipients of the company's emails will follow the advice contained in the body of at least some of the emails they receive. E) Some of the tax preparation company's clients would try to illegally evade penalties if they knew how to do so.

A) If the email doesn't elsewhere suggest that the client do anything illegal, then the company does not need legal protection. We were looking for a choice that confirms that the disclaimer offers no legal protection if the email doesn't suggest illegal activity. We need a confirmation that suggestions of illegal activity are the ONLY reason the company would need protection. B: Doesn't tell us what happens if the messages don't suggest illegal action. Therefore, we can't conclude that the disclaimer serves no purpose. C: Whether the recipients notice or ignore the disclaimer doesn't tell us anything about whether that disclaimer serves any purpose. D: Whether recipients follow advice in the emails doesn't tell us about whether the emails disclaimer serves any legal purpose for the company. E: Whether recipients would act illegally if they knew how to doesn't tell us anything about whether the disclaimer serves any purpose for the company.

Columnist: Contrary to what many people believe, the number of species on Earth is probably not dwindling. Extinction is a natural process, and about as many species are likely to go extinct this year as went extinct in 1970. But the emergence of new species is also a natural process; there is no reason to doubt that new species are emerging at about the same rate as they have been for the last several centuries. Which one of the following, if true, most weakens the columnist's argument? (practice test 10) A) In 1970 fewer new species emerged than went extinct. B) The regions of the world where new species tend to emerge at the highest rate are also where species tend to go extinct. C) The vast majority of the species that have ever existed are now extinct D) There is no more concern now about the extinction of species than there was in 1970 E) Scientists are now better able to identify species facing serious risk of extinction than they were in 1970

A) In 1970 fewer new species emerged than went extinct. If the number of species declined in 1970, then a comparison to that year would indicate that species will decline this year as well

Historian: The Land Party achieved its only national victory in Banestria in 1935. It received most of its support that year in rural and semirural areas, where the bulk of Banestria's population lived at the time. The economic woes of the years surrounding that election hit agricultural and small business interests the hardest, and the Land Party specifically targeted those groups in 1935. I conclude that the success of the Land Party that year was due to the combination of the Land Party's specifically addressing the concerns of these groups and the depth of the economic problems people in these groups were facing. Each of the following, if true, strengthens the historian's argument EXCEPT: (LSATMax test) A) In preceding elections the Land Party made no attempt to address the interests of economically distressed urban groups. B) Voters are more likely to vote for a political party that focuses on their problems. C) The Land Party had most of its successes when there was economic distress in the agricultural sector. D) No other major party in Banestria specifically addressed the issues of people who lived in semirural areas in 1935. E) The greater the degree of economic distress someone is in, the more likely that person is to vote.

A) In preceding elections the Land Party made no attempt to address the interests of economically distressed urban groups. Keyword: urban vs "suburban" and "rural" in the passage

Advertisement: In a carefully controlled study, blindfolded volunteers were divided evenly into five groups. Each volunteer tasted Sparkle Cola and one of five competing colas, each group tasting a different cola. Most of the volunteers said they preferred Sparkle Cola to the competing cola tasted. This shows that Sparkle Cola elicits a more favorable response from consumers than any of the competing colas tested. The reasoning in the advertisement is most vulnerable to criticism on which of the following grounds? (practice test 3) A) It overlooks the possibility that a generalization true of the entire group was not true of each of the five smaller groups B) It takes for granted that most of the volunteers would buy Sparkle Cola rather than one of the other colas tasted, at least in situations where Sparkle Cola is not much more expensive C) It overlooks the possibility that some cola not tested in the study would have elicited a more favorable response than Sparkle Cola D) It overlooks the possibility that many people may prefer Sparkle Cola to competing colas for reasons such as the packaging or price of Sparkle Cola, rather than its taste E) It is based on a study that does not elicit consumers' responses to any beverages other than colas

A) It overlooks the possibility that a generalization true of the entire group was not true of each of the five smaller groups Suppose that four of the five groups chose Sparkle Cola by a tiny margin, but the fifth group unanimously preferred the competitor. It would be wrong to conclude that Sparkle Cola is better than any competitor C: Information about sodas not tested has no impact on the argument

Max: Although doing so would be very costly, humans already possess the technology to build colonies on the Moon. As the human population increases and the amount of unoccupied space available for constructing housing on Earth diminishes, there will be a growing economic incentive to construct such colonies to house some of the population. Thus, such colonies will almost certainly be built and severe overcrowding on Earth relieved. Max's argument is most vulnerable to criticism on which of the following grounds? (practice test 5) A) It takes for granted that the economic incentive to construct colonies on the moon will grow sufficiently to cause such a costly project to be undertaken B) It takes for granted that the only way of relieving severe overcrowding on earth is the construction of colonies on the moon C) It overlooks the possibility that colonies will be built on the moon regardless of any economic incentive to construct such colonies to house some of the population D) It overlooks the possibility that colonies on the moon might themselves quickly become overcrowded E) It takes for granted that none of the human population would prefer to live on the moon unless earth were seriously overcrowded

A) It takes for granted that the economic incentive to construct colonies on the moon will grow sufficiently to cause such a costly project to be undertaken Max cites that there will be "growing" economic incentive, but never that the economic incentive will be large enough to build on the moon E: Max doesn't assume that NO ONE would prefer to live on the moon except in the case of an overcrowded earth

The owners of Uptown Apartments are leaning toward not improving the apartment complex; they believe that the increased rents they could charge for improved apartments would not cover the costs of the improvements. But the improvements would make the surrounding housing, which they also own, more valuable and rentable for higher rents. So the owners should make the improvements. The reasoning in which of the following is most similar to the reasoning in the argument above? (Practice test 1) A) John's injured knee does not cause him a lot of pain, so he does not want to undergo the pain of surgery to heal it. But the surgery would enable him to exercise regularly again. Thus John should have the surgery. B) Since its fishing season lasts only six months, Laketown Fishing Company prefers renting boats to buying its own. But since boats can be used for other purposes during the fishing season, it has made the wrong decision. C) Max's mechanic thinks there is a crack in the left cylinder head of Max's car and wants to remove the engine to check. Such a diagnostic engine removal would cost about $175, even if the cylinder head does not need replacement. But if the cylinder head is cracked and is not replaced, the engine will be ruined. So Max should have the mechanic check for the crack. D) Because of dental problems, Leona cut her consumption of candy. Consequently, she learned to enjoy fruit more. Thus, dental problems, which can lead to other health problems, led in her case to an improved diet and better health overall. E) Bulk Fruit Company is deciding whether to market a new hybrid fruit. It is enthusiastic about the idea, since research suggests that people will come to like this fruit. Therefore, it is in the long-term interest of the company to market the hybrid fruit.

A) John's injured knee does not cause him a lot of pain, so he does not want to undergo the pain of surgery to heal it. But the surgery would enable him to exercise regularly again. Thus John should have the surgery. We can make a prediction that the answer will have a similar structure of a recommendation to do something because even though it doesn't serve one purpose, it serves a different purpose. B: This choice doesn't provide a recommendation but instead evaluates an already-established decision. C: This doesn't match because Max was never resistant to the idea of having the mechanic check for the crack. D: This choice provides a causal relationship that has already happened, and doesn't present a definite recommendation like the passage does. E: Doesn't match because the company is enthusiastic about the idea, not resisting it as in the passage.

History provides many examples of technological innovations being strongly resisted by people whose working conditions without those innovations were miserable. This shows that social inertia is a more powerful determinant of human behavior than is the desire for comfort or safety. Which one of the following, if true, most seriously undermines the reasoning in the argument? (Practice Test 9) A) People correctly believe that technological innovations often cause job loss B) People are often reluctant to take on new challenges C) Some examples of technological innovation have been embraced by workers D) People tend to adapt easily to gradually implemented technological innovations E) People correctly believe that technological innovations almost always increase workers' productivity

A) People correctly believe that technological innovations often cause job loss If this is true, than it's possible that the proposed idea of "social inertia" has nothing to do with the resistance to technological innovations

Editor: Many candidates say that if elected they will reduce governmental intrusion into voters' lives. But voters actually elect politicians who instead promise that the government will provide assistance to solve their most pressing problems. Governmental assistance, however, costs money, and money can come only from taxes, which can be considered a form of governmental intrusion. Thus, governmental intrusion into the lives of voters will rarely be substantially reduced over time in a democracy. Which one of the following, if true, would most strengthen the editor's argument? (LSAT Max Practice Test) A) Politicians who win their elections usually keep their campaign promises. B) Politicians never promise what they really intend to do once in office. C) The most common problems people have are financial problems. D) Governmental intrusion into the lives of voters is no more burdensome in nondemocratic countries than it is in democracies. E) Politicians who promise to do what they actually believe ought to be done are rarely elected.

A) Politicians who win their elections usually keep their campaign promises. On the LSAT, whenever a premise is based on what someone says, there's a good chance that the argument will erroneously rely on that being true. There's a gap between what someone says and what someone does, so to strengthen this argument we look for an answer that connects those dots.

If Skiff's book is published this year, Professor Nguyen vows she will urge the dean to promote Skiff. Thus, if Skiff's book is as important and as well written as Skiff claims, he will be promoted, for Nguyen will certainly keep her promise, and the dean will surely promote Skiff if Nguyen recommends it. The argument's conclusion can be properly inferred if which one of the following is assumed? (Practice test 4) A) Skiff's book will be published this year if it is as important as he claims it is. B) Skiff needs to publish a book before he can be promoted C) Professor Nguyen believes that Skiff's book is well written D) Skiff's book will not be published unless it is as important and as well written as he claims it is. E) Skiff will not be promoted unless Professor Nguyen urges the dean to do so

A) Skiff's book will be published this year if it is as important as he claims it is. The support is that IF Skiff's book is important and well-written, Skiff's book will be published; if Skiff's book is published, Nguyen will recommend Skiff for promotion which will lead to Skiff being promoted. Our prediction follows from the statement in this choice that being "important" is sufficient to get the book published D: confuses a sufficient condition for Skiff's book to be published with a necessary condition

Terrence Gurney suggests that because his books appeal to a wide audience, he is not given due credit for his literary achievements. Surely he is mistaken. Gurney's books tell interesting stories, but the writing is flat, leaving no lasting impression on the reader. This is likely the reason that Gurney has not received praise for literary achievement. Which one of the following most accurately states the argument's overall conclusion? (practice test 1) A) Terrence Gurney is mistaken when he suggests that the wide appeal of his books has prevented him from being given due credit for his literary achievements. B) Terrence Gurney's books are not significant literary achievements. C) Even though Terrence Gurney's books tell interesting stories, his writing is flat and leaves no lasting impression on the reader. D) Terrence Gurney has not been given due credit for his literary achievements because his books appeal to such a wide audience E) Terrence Gurney should have received some praise for his literary achievements despite the fact that his writing is flat and leaves no lasting impression on the reader.

A) Terrence Gurney is mistaken when he suggests that the wide appeal of his books has prevented him from being given due credit for his literary achievements. Main conclusion, represented by statement "Surely he is mistaken." B: Author never claims that the books aren't significant literary achievements. C: Represents arguer's support, not conclusion D: represents Gurney's own conclusion and support, not the arguer's E: Contradicts the argument

TV meteorologist: Our station's weather forecasts are more useful and reliable than those of the most popular news station in the area. After all, the most important question for viewers in this area is whether it will rain, and on most of the occasions when we have forecast rain for the next day, we have been right. The same cannot be said for either of our competitors. Which one of the following, if true, most strengthens the meteorologist's argument? (Practice Test 5) A) The meteorologist's station forecast rain more often than did the most popular news station in the area B) The less popular of the competing stations does not employ any full-time meteorologists C) The most popular news station in the area is popular because of its investigative news reports D) The meteorologist's station has a policy of not making weather forecasts more than three days in advance E) On most of the occasions when the meteorologist's station forecast that it would not rain, at least one of its competitors also forecast that it would not rain

A) The meteorologist's station forecast rain more often than did the most popular news station in the area B: no impact on the argument

A large survey of scientists found that almost all accept Wang's Law, and almost all know the results of the Brown-Eisler Experiment. But those results together with Wang's Law contradict the Minsk Hypothesis. Therefore, most of the scientists surveyed reject the Minsk Hypothesis. The argument requires assuming which of the following? (practice test 7) A) The scientists surveyed are generally aware that the results of the Brown-Eisler Experiment together with Wang's Law contradict the Minsk Hypothesis. B) The scientists in the survey who know the results of the Brown-Eisler Experiment are exactly the same ones who accept Wang's Law C) Almost all of the scientists surveyed are familiar with the way in which the results of the Brown-Eisler Experiment were obtained D) The sample is large enough to be representative of scientists in the field. E) Wang's Law has in fact been shown to be true.

A) The scientists surveyed are generally aware that the results of the Brown-Eisler Experiment together with Wang's Law contradict the Minsk Hypothesis. If the scientists surveyed weren't generally aware of this contradiction, then the contradiction wouldn't tell us anything about what the scientists would reject. B: Too strong to be necessary to the argument. We must assume that there is SOME overlap between the scientists who know the results and who accept the Law, but we don't need to assume they are EXACTLY the same ones

Voting records regularly show that people over 65 vote in the highest percentages while young adults are least likely to vote. This indicates that citizens are becoming increasingly disconnected from the political system with each passing generation. The argument's reasoning is questionable in that the argument (practice test 8) A) compares an early stage of one's generation to a later stage of another B) fails to take into account the relative sizes of the generations compared C) provides evidence for a phenomenon without providing an explanation of that phenomenon D) confuses the cause of an effect with the effect itself E) overlooks the possibility that voting patterns among age groups will change in the future

A) compares an early stage of one's generation to a later stage of another These two groups aren't similar enough to be compared in a way that can support the argument's conclusion

Journalism's purpose is to inform people about matters relevant to the choices they must make. Yet, clearly, people often buy newspapers or watch television news programs precisely because they contain sensationalistic gossip about people who they will never meet and whose business is of little relevance to their lives. Obviously, then, the sensationalistic gossip contained in newspapers and television news programs _____________. Which one of the following most logically completes the argument? (Practice test 6) A) is at least sometimes included for nonjournalistic reasons B) prevents those news media from achieving their purpose C) is more relevant to people's lives now than it used to be D) should not be thought of as a way of keeping an audience entertained E) is of no value to people who are interested in journalism

A) is at least sometimes included for nonjournalistic reasons. If newspapers and TV programs also contain info (gossip) that doesn't affect readers' lives, then it follows that the gossip doesn't fulfill the journalistic purpose. Choice B: too strong to complete the argument - even though the gossip doesn't contribute to the purpose of news, it doesn't hinder it since it isn't the only content being aired. Choice C: Irrelevant to the argument Choice D: Whether gossip is a way of keeping an audience entertained or not isn't a factor in the argument. Choice E: too strong. gossip doesn't have to have NO VALUE to be not fulfilling to a journalistic purpose.

Newspaper subscriber: Arnot's editorial argues that by making certain fundamental changes in government we would virtually eliminate our most vexing social ills. But clearly this conclusion is false. After all, the argument Arnot makes for this claim depends on the dubious assumption that government can be trusted to act in the interest of the public. Which one of the following most accurately expresses a flaw in the argument's reasoning? (practice test 9) A) it repudiates a claim merely on the grounds that an inadequate argument has been given for B) it treats a change that is required for virtual elimination of society's most vexing social ills as a change that will guarantee the virtual elimination of those ills C) it fails to consider that, even if an argument's conclusion is false, some of the assumptions used to justify that conclusion may nonetheless be true D) it distorts the opponent's argument and then attacks this distorted argument E) it uses the key term "government" in one sense in a premise and in another sense in the conclusion

A) it repudiates a claim merely on the grounds that an inadequate argument has been given for It may be that Arnot's argument isn't well constructed, but it doesn't mean that Arnot is wrong

Political scientist: People become unenthusiastic about voting if they believe that important problems can be addressed only by large numbers of people drastically changing their attitudes and that such attitudinal changes generally do not result from government action. The decreasing voter turnout is thus entirely due to a growing conviction that politicians cannot solve the most important problems. The reasoning in the political scientist's argument is most vulnerable to criticism on the grounds that the argument (Practice test 2) A) presumes, without providing justification, that there is no cause of decreasing voter turnout other than the belief that few important problems can be solved by government action. B) presumes, without providing justification, that there are no political solutions to the most important problems C) infers that important problems can be seriously addressed if people's attitudes do change from the premise that these problems cannot be addressed if people's attitudes do not change D) undermines its claim that people no longer believe there are political solutions to important problems by suggesting that people are dissatisfied with politicians E) presumes, without providing justification, that voter apathy prevents the attitudinal changes that result in finding solutions to important problems.

A) presumes, without providing justification, that there is no cause of decreasing voter turnout other than the belief that few important problems can be solved by government action. Simplify the stimulus: "People don't vote if they believe (1) attitude change is what fixes problems and (2) gov action can't change people's attitudes, so (3) politicians can't solve important problems. Therefore, low voter turnout is ENTIRELY due to this belief" Political scientist presents a cause for decreasing voter turnout and proceeds to argue that it is the ONLY possible cause. B: Political scientist doesn't make this assumption C: Error in conditional reasoning (if NOT A, then NOT B; therefore, if A then B) D: The argument does not undermine its claim that people no longer believe in political solutions E: This assumption is outside the scope of the argument

The short-term and long-term interests of a business often conflict; when they do, the morally preferable act is usually the one that serves the long-term interest. Because of this, businesses often have compelling reasons to execute the morally preferable act. Which one of the following, if assumed, enables the conclusion of the argument to be properly drawn? (Practice test 5) A) A business's moral interests do not always provide compelling reasons for executing an act B) A business's long-term interests often provide compelling reasons for executing an act C) The morally preferable act for a business to execute and the long-term interests of the business seldom conflict D) The morally preferable act for a business to execute and the short-term interests of the business usually conflict E) When a business's short term and long term interests conflict, morality alone is rarely the overriding consideration

B) A business's long-term interests often provide compelling reasons for executing an act To test this answer, combine this statement with the argument's evidence to see if that leads to the argument's conclusion D: just restates the first part of the passage

Biologists found that off the northeast coast of a certain country the P-plankton population has recently dropped 10 percent. Additionally, fish species X, Y, and Z are beginning to show extraordinarily high death rates in the region. Since these species of fish are known to sometimes eat P-plankton, biologists believe the two phenomena are connected, but the exact nature of the connection is unknown. No other species in the ecosystem appear to be affected. Which one of the following, if true, most helps to explain the biologists' findings? (practice test 10) A) Several major pharmaceutical companies in the region have been secretly dumping large amounts of waste into the ocean for many years B) A new strain of bacteria is attacking P-plankton by destroying their cell walls and is attacking the respiratory systems of fish species X, Y, and Z C) A powerful toxin in the water is killing off P-plankton by inhibiting their production of a chemical they use in reproduction D) Fish species X, Y, and Z are all experiencing widespread starvation within the affected region, and the loss of P-plankton is driving their death rates up even higher E) Global warming has changed the climatic conditions of the ocean all along the northeast coast of the country

B) A new strain of bacteria is attacking P-plankton by destroying their cell walls and is attacking the respiratory systems of fish species X, Y, and Z D: Doesn't explain why P-plankton is dying in the first place

Literary critic: A folktale is a traditional story told in an entertaining way, which may lead some to think that folktales lack deeper meaning. But this is not the case. A folktale is passed along by a culture for generations, and each storyteller adds something of his or her own to the story, and in this way folktales provide great insight into the wisdom of the culture. The main conclusion of the literary critic's argument can be properly inferred if which one of the following is assumed? (practice test 4) A) Any tale that is passed along by a culture for generations can provide great insight into the wisdom of that culture. B) Any tale that provides insight into the wisdom of a culture is deeply meaningful in some respect C) Not every tale that lacks deep meaning or beauty is told solely for entertainment D) Any tale with deep meaning provides great insight into the wisdom of the culture by which it has been passed on E) A story that is told primarily for entertainment does not necessarily lack a deeper meaning

B) Any tale that provides insight into the wisdom of a culture is deeply meaningful in some respect Connects the premises to the conclusion. Folktales are insight into the wisdom of a culture, which are deeply meaningful. Therefore, folktales are deeply meaningful A: States an inference that we can draw based on provided information, not an assumption that would help the critic's conclusion

Carpal tunnel syndrome, a nerve disorder that affects the hands and wrists, is often caused by repetitive motions such as typing on a keyboard. A recent study of office workers found that, among those who do similar amounts of typing, workers reporting the least control over their own work had almost three times the risk of developing carpal tunnel syndrome as did those who reported the most control. Which one of the following, if true, most helps to explain the study's findings? (practice test 1) A) Office workers who have the most control over their own work tend to do significantly less typing than do those who have the least control over their own work. B) Feeling a lack of control over one's own work tends to put one under emotional stress that makes one more susceptible to nerve disorders. C) The keyboards on which office workers type tend to put typists' arms and hands in positions that promote the development of carpal tunnel syndrome. D) Among office workers who rarely use keyboards, the rate of carpal tunnel syndrome is much higher for those who feel that they lack control over their work. E) Office workers who have the most control over their own work tend to perform repetitive motions other than typing more often than do office workers with the least control over their work.

B) Feeling a lack of control over one's own work tends to put one under emotional stress that makes one more susceptible to nerve disorders. A and D: This is not in line with the stimulus that says the workers did "similar amounts of writing" C: If keyboards are problematic, they'd be problematic for everyone E: This choice is counterintuitive

In most industrial waste products that contain the toxic chemical XTX, the concentration of this chemical is approximately 1,000 parts per million. A federal law intended to reduce the harm that can result from the introduction of XTX into the environment permits a company to dispose of these waste products in a dump for hazardous waste, but only if the concentration of XTX is below 500 parts per million. Waste products with concentrations above that level must be destroyed by incineration. The law further specifies that manufacturers may not dilute XTX-containing waste products to bring their concentration of XTX down to a permissible level for dumping. Which one of the following, if true, argues most strongly for the inclusion of the antidilution provision of the law? (practice test 3) A) If improperly incinerated, waste products containing undiluted concentrations of XTX can release into the environment a gaseous form of the chemical that is more than twice as toxic as XTX is in its usual liquid state B) If present in the environment in sufficient quantities, the diluted XTX is as harmful as the more concentrated XTX C) When XTX is exposed to sunlight and oxygen, it eventually breaks down into a number of components that individually and collectively carry no risk of environmental harm D) Most owners of dumps for hazardous waste are willing to accept XTX for disposal in their facilities only in concentrations below 800 parts per million E) To manufacturers, the cost of diluting and disposing of waste products containing XTX is approximately the same as the cost of destroying these products by incineration.

B) If present in the environment in sufficient quantities, the diluted XTX is as harmful as the more concentrated XTX If it is the case that diluted XTX can be as harmful as concentrated, it makes sense to include an antidilution provision in the law.

Politician: Democracy requires that there be no restrictions on the ability of citizens to share their ideas freely, without fear of reprisal. Therefore the right to have private conversations, unmonitored by the government, is essential to democracy. For a government to monitor conversations on the Internet would thus be a setback for democracy. Which one of the following most accurately describes the role played in the argument by the claim that democracy depends on the ability of citizens to share their ideas freely, without fear of reprisal? A) It is a claim for which no support is provided, and which is used to support only the argument's main conclusion. B) It is a claim for which no support is provided, and which is used to support a claim that is itself used to support the argument's main conclusion. C) It is a claim for which support is provided, and which is in turn used to support the argument's main conclusion D) It is the argument's main conclusion and is inferred from two other statements in the argument, one of which is used to support the other E) It is the argument's main conclusion and is inferred from two other statements in the argument, neither of which is used to support the other

B) It is a claim for which no support is provided, and which is used to support a claim that is itself used to support the argument's main conclusion. Logical indicator words (because, since, thus, therefore, etc) can be quite helpful in determining the intended relationship between ideas. A: First half works, second doesn't; it supports the intermediate conclusion, not the overall conclusion

Astrologer: Although some scientists have claimed that there is no correlation between people's astrological signs and their personality types, this claim is scientifically unjustified. Since science does not have precise criteria for distinguishing one personality type from another, scientific studies cannot be used to disprove a correlation between personality type and any other phenomenon. Which one of the following most accurately describes the role played in the astrologer's argument by the statement that scientific studies cannot be used to disprove a correlation between personality type and any other phenomenon? (practice test 2) A) It is a claim offered as support for a conclusion that is in turn offered as support for the overall conclusion drawn in the argument B) It is a conclusion for which support is offered and that in turn is offered as support for the overall conclusion drawn in the argument C) It is the overall conclusion drawn in the argument D) It summarizes a position that the argument as a whole is directed toward discrediting E) It provides a specific instance of the general principle that the argument as a whole is directed toward establishing

B) It is a conclusion for which support is offered and that in turn is offered as support for the overall conclusion drawn in the argument A: The claim in question is not SUPPORTING an intermediate conclusion, rather it IS an intermediate conclusion C: The referenced claim is not the overall conclusion of the argument , which is that "this claim is scientifically unjustified" D: Statement in question does not summarize a position that the argument as a whole is directed toward discrediting E: Referenced statement is not a SPECIFIC instance and the argument isn't seeking to illustrate a general principle

Singletary: We of Citizens for Cycling Freedom object to the city's new ordinance requiring bicyclists to wear helmets. If the city wanted to become a safer place for cyclists, it would not require helmets. Instead, it would construct more bicycle lanes and educate drivers about bicycle safety. Thus, passage of the ordinance reveals that the city is more concerned with the appearance of safety than with bicyclists' actual safety. Which one of the following most accurately describes the role played in Singletary's argument by the statement that mentions driver education? (practice test 5) A) It is cited as evidence for the claim that the city misunderstands the steps necessary for ensuring bicyclists' safety B) It is used as partial support for a claim about the motivation of the city C) It is offered as evidence of the total ineffectiveness of the helmet ordinance D) It is offered as an example of further measures the city will take to ensure bicyclists' safety E) It is presented as an illustration of the city's overriding interest in the public image

B) It is used as partial support for a claim about the motivation of the city This statement is PART of the reason that Singletary believes the city cares about appearance more than safety A: This choice mischaracterizes the claim that the statement is evidence for

Medical reporter: Studies have consistently found that taking an aspirin a day thins the blood slightly, thereby helping to prevent or reduce the severity of heart disease. Since heart disease is one of the most common types of ill health in industrialized nations, most people in such nations would therefore be in better health if they took an aspirin a day. The reasoning in the doctor's argument is most vulnerable to criticism on which one of the following grounds? (Practice test 4) A) It takes for granted that if medication can reduce the severity of heart disease, it can also prevent some cases of heart disease. B) It overlooks the possibility that even if a disease is one of the most common in a nation, most people in that nation are not in significant danger of developing that disease. C) It overlooks the possibility that preventing or reducing the severity of heart disease has little or no effect on any of the other most common diseases in industrialized nations. D) It fails to address the possibility that taking an aspirin a day is not the single most effective measure for preventing heart disease. E) It fails to address the possibility that the studies on the beneficial effects of aspirin were only conducted in industrialized nations

B) It overlooks the possibility that even if a disease is one of the most common in a nation, most people in that nation are not in significant danger of developing that disease. Although heart disease is one of the most common in the nation, it may still affect only a tiny fraction of the population in such nations. C: Even if reducing the severity of heart disease has no effect on any other disease, it can still make one healthier

In 1996, all ResearchTech projects were funded either by the government or by private corporations. The Gilman Survey, a ResearchTech project, was not funded by the government but was conducted in 1996. It must therefore have been funded by private corporations. Which one of the following is most similar in its reasoning to the argument above? (practice test 10) A) Legal restrictions on consumer purchases have a variety of aims; for example, some are paternalistic, and others are designed to protect civil liberties. Ordinance 304, a legal restriction on alcohol sales, does not protect civil liberties. It must therefore be paternalistic. B) Legal restrictions on consumer purchases, such as Ordinance 304, are either paternalistic or protect civil liberties. Ordinance 304 is not paternalistic, so it must provide civil liberties. C) Ordinance 304 is not paternalistic. Since all legal restrictions on consumer purchases are either paternalistic or designed to protect the environment, the purpose of Ordinance 304 must not be to protect the environment D) Legal restrictions on consumer purchases are either paternalistic or designed to protect civil liberties. All ordinances passed in 1993 are paternalistic. Since Ordinance 304 was passed in 1993, it must be a legal restriction on consumer purchases E) Ordinance 304 should be exercised only in order to protect civil liberties or to protect consumers from self-harm. The mayor's last exercise of Ordinance 304 does not protect civil liberties, so it must have been intended to protect consumers from self-harm

B) Legal restrictions on consumer purchases, such as Ordinance 304, are either paternalistic or protect civil liberties. Ordinance 304 is not paternalistic, so it must provide civil liberties.

Human skin gives off an array of gaseous substances, including carbon dioxide and lactic acid, both of which attract mosquitoes. However, neither of these two substances, whether alone or combined with one another, will attract mosquitoes as much as a bare human arm will, even in complete darkness, where a mosquito has no visual cues. Therefore, some other gaseous substance given off by human skin also attracts mosquitoes. The reasoning in the argument requires which one of the following assumptions? (Practice test 8) A) Mosquitoes do not communicate with one another. B) Mosquitoes are not attracted to humans by body heat. C) Human skin gives off gaseous substances in greater amounts during the day than during the night D) Mosquitoes are no more successful in finding a bare human arm in darkness than in light E) Human skin never gives off any gaseous substances that repel mosquitoes

B) Mosquitoes are not attracted to humans by body heat. Gives an alternative reason why mosquitoes are attracted to the human arm instead of to the gaseous substances

Company president: Most of our best sales representatives came to the job with a degree in engineering but little or no sales experience. Thus, when we hire sales representatives, we should favor applicants who have engineering degrees but little or no sales experience over applicants with extensive sales experience but no engineering degrees. Which one of the following, if true, most seriously weakens the company president's argument? (practice test 4) A) Some of the company's sales representatives completed a degree in engineering while working for the company. B) Most of the people hired by the company as sales representatives have had a degree in engineering but no sales experience. C) Most of the customers that the company's representatives work with have a degree in engineering D) Most of the people who apply for a sales representative position with the company do not have a degree in engineering E) Some of the people who the company has hired as sales representatives and who were subsequently not very good at the job did not have extensive previous sales experience

B) Most of the people hired by the company as sales representatives have had a degree in engineering but no sales experience. The answer suggests that there is no correlation between job performance and qualifications. This choice debunks the causal relationship between having an engineering degree and being successful, which makes the conclusion more far fetched A: Just because some of the sales reps completed their degrees while on the job doesn't mean that a degree is not a desirable qualification for applicants. We also don't know if these sales reps are among the best

Art historian: This painting, purportedly by Mary Cassatt, is a forgery. Although the canvas and other materials are consistent with most of Cassatt's work, and the subject matter is similar to that of Cassatt's finest paintings, the brush style of this painting is not found in any work known to be Cassatt's. Hence this painting is definitely not a genuine Cassatt. The art historian's argument depends on assuming which of the following? (practice test 2) A) The type of canvas and other materials that Cassatt used in most of her work were readily available to others. B) None of Cassatt's works are painted using a brush style that is not exhibited in any of her known works. C) Cassatt's work generally had a characteristic subject matter that distinguished it from the work of other painters in her era. D) The most characteristic feature of Cassatt's work is her brush style. E) No painter other than Cassatt would be able to match Cassatt's brush style perfectly.

B) None of Cassatt's works are painted using a brush style that is not exhibited in any of her known works. Test this choice by considering the opposite case; because the opposite weakens the argument, we know we are dealing with a required assumption. A: author makes no assumptions about what it actually takes to make a forgery. C: Not relevant to the differing brush styles D: not necessary for the brush style to be the MOST characteristic feature; necessary assumptions generally avoid exaggerated claims E: Does not assume that NO ONE can match the style perfectly. The logical opposite of this choice does not weaken the argument, so it's not the necessary assumption

Ethicist: On average, animals raised on grain must be fed sixteen pounds of grain to produce one pound of meat. A pound of meat is more nutritious for humans than a pound of grain, but sixteen pounds of grain could feed many more people than could a pound of meat. With grain yields leveling off, large areas of farmland going out of production each year, and the population rapidly expanding, we must accept the fact that consumption of meat will soon be morally unacceptable. Which one of the following, if true, would most weaken the ethicist's argument? (LSAT Max Practice Test) A) Even though it has been established that a vegetarian diet can be healthy, many people prefer to eat meat and are willing to pay for it. B) Often, cattle or sheep can be raised to maturity on grass from pastureland that is unsuitable for any other kind of farming. C) If a grain diet is supplemented with protein derived from non-animal sources, it can have nutritional value equivalent to that of a diet containing meat. D) Although prime farmland near metropolitan areas is being lost rapidly to suburban development, we could reverse this trend by choosing to live in areas that are already urban. E) Nutritionists agree that a diet composed solely of grain products is not adequate for human health.

B) Often, cattle or sheep can be raised to maturity on grass from pastureland that is unsuitable for any other kind of farming. Any answer that raises a consideration suggesting that there will be times when it's fine to eat meat would weaken the conclusion that we need to cut it out completely. If we can find one situation to undermine a strong conclusion, then we've got our answer!

Dried parsley should never be used in cooking, for it is far less tasty and healthful than fresh parsley is. Which one of the following principles, if valid, most clearly helps to justify the argument above? (practice test 2) A) Fresh ingredients should be used in cooking whenever possible. B) Only the tastiest ingredients should ever be used in cooking. C) Ingredients that should never be used in cooking are generally neither tasty nor healthful D) Parsley that is not both tasty and healthful should never be used in cooking. E) In cooking, dried ingredients are inferior to fresh ingredients.

B) Only the tastiest ingredients should ever be used in cooking. As long as the principle specifies A necessary condition for cooking that dried parsley fails to meet, then the conclusion follows. A: Not strong enough due to the phrase "whenever possible." C and D: The stimulus doesn't say that dried parsley isn't tasty or healthful, just that its less so than fresh parsley E: This is a factual statement not a principle. Tells us nothing about the conditions that should be met in order for an ingredient to be used in cooking

The television network's advertisement for its new medical drama grossly misrepresents what that program is like. Thus, it will not as effectively attract the sort of viewers likely to continue watching the program as would the advertisement that the program's producers favored; people who tune in to the first episode based on false expectations will be unlikely to watch subsequent episodes. The argument relies on which of the following assumptions? (practice test 3) A) Most viewers who tune in to the first episode of the program will do so because of the network's advertisement for the program B) The advertisement that the program's producers favored would not have grossly misrepresented what the program would be like C) Most people who tune in to the first episode of the program and become loyal viewers will not have tuned in to the first episode as a result of the network's advertisement for the program D) If the advertisement that the program's producers favored were used instead of the network's advertisement, almost all of the viewers who tuned in to the first episode would tune in to subsequent episodes as well E) Most people who become loyal viewers of a program do not miss the program's first episode

B) The advertisement that the program's producers favored would not have grossly misrepresented what the program would be like In order for the argument's conclusion to follow, we must assume that the producer's advertisement also does not grossly misrepresent the program D: supports the claim, but it's too strong to be assumed

Though Earth's human population is increasing, it currently uses only a relatively small fraction of the supply of fresh water. Thus, claims that water shortages will plague humankind in the near future unless population growth trends change are simply mistaken. Which one of the following, if true, most seriously weakens the argument above? (practice test 3) A) Population growth trends are notoriously hard to predict with reasonable accuracy B) The amount of fresh water available to meet the needs of Earth's population varies significantly from region to region C) Not all of Earth's population will adopt water conservation methods in the near future D) If Earth's population continues to increase, it will eventually outstrip all available resources E) The percentage of fresh water used for agriculture is likely to grow more quickly than is the percentage used for industry

B) The amount of fresh water available to meet the needs of Earth's population varies significantly from region to region Even if the Earth OVERALL uses only a small fraction of fresh water supply, there could be serious water shortages in specific regions which could still cause water shortages to plague humankind in the future D: Stimulus discusses the water shortages that may occur in the "near future," while this prompt discusses what will "eventually" happen

Biologist: Some computer scientists imagine that all that is required for making an artificial intelligence is to create a computer program that encapsulates the information contained in the human genome. They are mistaken. The operation of the human brain is governed by the interactions of proteins whose structures are encoded in the human genome. Which one of the following is an assumption required by the biologist's argument? (Practice test 7) A) The functions of the human brain are governed by processes that cannot be simulated by a computer B) The interactions of the proteins that govern the operation of the human brain are not determined by the information contained in the human genome C) The only way to create an artificial intelligence is to model it on the operation of the human brain. D) The amount of information contained in the human genome is too large to be easily encapsulated by a computer program E) It is much more difficult to write a program that encapsulates the interactions of proteins than to write a program that encapsulates the information contained in the human genome

B) The interactions of the proteins that govern the operation of the human brain are not determined by the information contained in the human genome If we negate this choice and say that the interactions of the proteins ARE determined by the information contained in the human genome, then this argument wouldn't work C: Whether or not there are ways to make an AI that isn't modeled on the human brain is irrelevant because the argument is discussing simply copying the information in the human brain to make an AI

After a nuclear power plant accident, researchers found radioactive isotopes of iodine, tellurium, and cesium—but no heavy isotopes—in the atmosphere downwind. This material came either from spent fuel rods or from the plant's core. Spent fuel rods never contain significant quantities of tellurium isotopes. Radioactive material ejected into the atmosphere directly from the core would include heavy isotopes. After the accident, steam, which may have been in contact with the core, was released from the plant. The core contains iodine, tellurium, and cesium isotopes, which are easily dissolved by steam. Of the following statements, which one is most strongly supported by the information above? (practice test 8) A) Radioactive material ejected into the environment directly from a nuclear power plant's core would not include tellurium isotopes B) The radioactive material detected by the researchers was carried into the atmosphere by the steam that was released from the plant C) The nuclear power plant's spent fuel rods were not damaged D) The researchers found some radioactive material from spent fuel rods as well as some material that was ejected into the atmosphere directly from the plant's core E) Spent fuel rods do not contain heavy isotopes in significant quantities

B) The radioactive material detected by the researchers was carried into the atmosphere by the steam that was released from the plant We are told that the only two places the material could have come from was either the spent fuel rods or the plant's core

Radio producer: Our failure to attract new listeners over the past several years has forced us to choose between devoting some airtime to other, more popular genres of music, and sticking with classical music that appeals only to our small but loyal audience. This audience, however loyal, did not generate enough advertising revenue for us to pay our bills, so if we appeal to them alone, our station risks going out of business. We should not take that risk. We should, therefore, devote some airtime to other, more popular genres of music. Which one of the following arguments is most similar in its pattern of reasoning to that used by the radio producer? (practice test 2) A) We should either buy blinds for the windows or make full-length curtains. Blinds would be very expensive to purchase. Thus, if cost is our greatest concern, we should make curtains. B) We should either make curtains for the windows or buy blinds. Since the windows are not standard sizes, if we buy blinds we will have to special order them. Since we do not have time to wait for special orders, we should make the curtains. C) For the living room windows, we can make curtains or valances or both. We want to have privacy; and while curtains provide privacy, valances do not. So we should make curtains but not valances. D) Since we have very little fabric, we will have to either buy more, or make valances instead of curtains. However, if we use this fabric to make valances, then we will have to buy blinds. Since it would be hard to buy fabric that matches what we already have, we should buy blinds. E) We should either buy blinds or make curtains for the windows. If we buy blinds but do not make valances, the windows will look bare. We should not have bare windows. So if we do not make the curtains, we should make the valances.

B) We should either make curtains for the windows or buy blinds. Since the windows are not standard sizes, if we buy blinds we will have to special order them. Since we do not have time to wait for special orders, we should make the curtains. To resolve a problem, the radio producer faces a dilemma between two courses of action. But one of these actions leads to an undesirable consequence; therefore, the radio producer concludes that they should pursue the alternative course of action. A: Outlines a dilemma, but presents a conclusion that uses conditional reasoning C: pursuing BOTH choices is not an option in the radio producer's argument D: Outlines a dilemma but then concludes that a third course of action would be better. No parallel to this in original stimulus E: Outlines a dilemma but then concludes that a third course of action would be better. No parallel to this in original stimulus. Also uses conditional reasoning

Some have argued that body size influences mating decisions throughout all societies. Their argument rests largely on self-reports of university-age students and on analyses of personal advertisements in newspapers for dating partners. The reasoning in the argument described above is the most vulnerable to criticism on the grounds that the argument (practice test 7) A) concludes that one kind of event causes another kind of event without ruling out the possibility that both kinds of events are the result of a third kind of event B) bases a conclusion on a sample that may be unrepresentative of the population about which the conclusion is drawn C) concludes that an effect has only one cause in the face of evidence that the effect has multiple causes D) uses a claim that applies only to entire societies to draw a conclusion about individual persons E) draws a universal conclusion on the basis of a very small number of individual cases.

B) bases a conclusion on a sample that may be unrepresentative of the population about which the conclusion is drawn Argument's support concerns only a narrow sample of people, which may not be representative of the population the argument's conclusion concerns (all societies) E: we can't infer that the number of individual cases is "very small." Sample could, based on the description we are given, include a great number of reports from a limited sample

Pundit: For many high school graduates, attending a university would be of no help in getting a corporate job. The attributes corporations value most in potential employees are initiative, flexibility, and the ability to solve practical problems. Many new high school graduates have these attributes already. The pundit's argument is most vulnerable to criticism on the grounds that it (Practice Test 7) A) fails to establish that university graduates do not have initiative, flexibility, and the ability to solve practical problems B) overlooks the possibility that corporations may require an attribute that potential employees can obtain only by attending a university C) provides no justification for the presumption that corporations only hire employees who have initiative, flexibility, and the ability to solve practical problems D) takes for granted that the only reason that high school grads go on to attend university is to improve their job prospects E) takes for granted that initiative, flexibility, and the ability to solve practical problems are attributes that can be acquired through study

B) overlooks the possibility that corporations may require an attribute that potential employees can obtain only by attending a university If this were true, then it would seem that university could help ALL high school grads get a corporate job C: Pundit doesn't make this presumption; uses term "highly likely," not ONLY

Alex: Shrimp farming results in damage to the environment, because investors make quick profits from such farming and then abandon the farms. Jolene: I disagree. Although some shrimp farms have proved unsustainable and have been quickly abandoned, properly built shrimp farms take a long time to construct and are costly to operate. Most owners try to make sure that their farms are productive for many years. Their dialogue provides the most support for the claim that Alex and Jolene disagree with each other over whether (practice test 6) A) most owners of shrimp farms eventually abandon their farms B) shrimp farming often yields a quick, easy profit C) shrimp farming hardly ever damages the environment D) abandonment of a shrimp farm results in damage to the environment E) some shrimp farmers are environmentally irresponsible

B) shrimp farming often yields a quick, easy profit Alex says shrimp farming yields a quick, easy profit whereas Jolene says that shrimp farms take a long time to build and are expensive to operate. A: Prompt has no opinion on whether farms are EVER abandoned, only whether they are abandoned QUICKLY C, D, and E: Jolene has no opinion on environmental issues

Pauline: Some environmentalists claim that for the salmon to be saved, the hydroelectric dams on the river must be breached. But if the dams are breached, given the region's growing population and booming industry, electrical costs will skyrocket. Roger: The dams are already producing electricity at optimal capacity. So regardless of whether they are breached, we will have to find additional energy sources for the region. The dialogue provides the most support for the claim that Pauline and Roger agree that (Practice test 4) A) production from other energy sources cannot be increased in the near future to compensate for electricity production lost by breaching the dams B) there will be no significant decrease in demand for electricity in the region in the near future C) if the dams remain in service but do not operate at optimal capacity, electrical costs in the region will rise D) some environmentalists who advocate saving the salmon believe that that goal overrides concerns about electrical costs E) finding additional energy sources will not decrease the electrical costs in the region

B) there will be no significant decrease in demand for electricity in the region in the near future D: Roger never gives an opinion on this

Teresa: If their goal is to maximize profits, film studios should concentrate on producing big-budget films rather than small-budget ones. For, unlike big-budget films, small-budget films never attract mass audiences. While small-budget films are less expensive to produce and, hence, involve less risk of unprofitability than big-budget films, low production costs do not guarantee the highest possible profits. Which of the following is an assumption required by Teresa's argument? (practice test 5) A) Each big-budget film is guaranteed to attract a ass audience B) A film studio cannot make both big budget films and small budget films C) A film studio will not maximize its profits unless at least some of the films attract mass audiences D) It is impossible to produce a big budget film in a financially efficient manner E) A film studio's primary goal should be to maximize profits

C) A film studio will not maximize its profits unless at least some of the films attract mass audiences If attracting mass audiences WEREN'T necessary to maximizing profits, then the argument for big budget films would be weakened A: Too extreme to be required by the passage

Any government practice that might facilitate the abuse of power should not be undertaken except in cases in which there is a compelling reason to do so. The keeping of government secrets is one such practice. Though government officials are sometimes justified in keeping secrets, too often they keep secrets for insubstantial reasons, and in so doing they wind up enabling abuses of power. When government officials conceal from the public the very fact that they are keeping a secret, this practice opens up even greater opportunity for abuse. Which one of the following can be properly inferred from the statements above? (practice test 8) A) in most cases in which government officials conceal information from the public, they are not justified in doing so B) In those cases in which government officials have a compelling reason to keep a secret, doing so does not facilitate an abuse of power C) A government official who justifiably keeps a secret should not conceal its existence without having a compelling reason to do so D) Government officials who conceal information without a compelling reason are thereby guilty of an abuse of power E) Government officials should keep information secret only if doing so does not make it easier for those officials to abuse their power

C) A government official who justifiably keeps a secret should not conceal its existence without having a compelling reason to do so Combine the first and last sentences in the stimulus to infer this choice

A study at a company found that most meetings showed diminishing returns after 30 minutes, and little could be expected after 60 minutes. Moreover, the most productive meetings were those for which a clear time frame was established. For a meeting at the company to achieve maximum productivity, then, it needs to have a clear time frame and be no more than 30 minutes long. Which one of the following most accurately expresses the conclusion drawn in the argument? (Practice Test 8) A) In general, a meeting at the company that is no more than 30 minutes long and has a clear time frame will achieve maximum productivity B) Most meetings at the company show diminishing returns after 30 minutes, according to a study C) A meeting at the company will be maximally productive only if it has a clear time frame and lasts no more than 30 minutes D) According to a study, meetings at the company were the most productive when they had clear time frames E) A study of meetings at the company says that little productivity should be expected after the 60-minute mark

C) A meeting at the company will be maximally productive only if it has a clear time frame and lasts no more than 30 minutes Conclusion: For a meeting to achieve maximum productivity, it needs to have a clear time frame and be no more than 30 minutes A: this choice conflates a necessary requirement (30 minute limit) for a sufficient requirement

Principle: Only if a professor believes a student knowingly presented someone else's ideas without attribution should the professor make an official determination that the student has committed plagiarism. Application: It is not the case that Professor Serfin should make an official determination that Walters committed plagiarism in the term paper about Willa Cather that Walters wrote for Serfin's class. Which one of the following, if true, justifies the above application of the principle? (practice test 10) A) Professor Serfin does not have completely compelling evidence to conclude that Walters presented someone else's ideas as if they were his own in the term paper about Willa Cather. B) If Walters had realized that the main thesis of his term paper is identical to the main thesis of a book he had read, Walters would have attributed the idea to the book C) Although the main thesis of Walters's term paper is identical to that of a book that he did not cite, Professor Serfin is convinced that Walters did not knowingly try to pass anyone else's ideas off as his own D) Walters does not believe that Professor Serfin should make an official determination that he plagiarized E) Professor Serfin has no intention of making an official determination that Walters plagiarized in the class.

C) Although the main thesis of Walters's term paper is identical to that of a book that he did not cite, Professor Serfin is convinced that Walters did not knowingly try to pass anyone else's ideas off as his own

Principle: Employees of telemarketing agencies should never do anything that predisposes people to dislike the agencies' clients. Application: If an employee of a telemarketing agency has been told by a person the employee has called that he or she does not want to buy the product of a client of the agency, the employee should not try to talk that person into doing so. Which one of the following, if true, justifies the given application of the principle above? (practice test 1) A) Any employee of a telemarketing agency is likely to be able to determine whether trying to talk someone into buying the product of a client of the agency after the person has said that he or she does not want to will likely engender animosity towards the client. B) Some employees of telemarketing agencies are unlikely to be certain about whether trying to talk someone into buying the product of a client of the agency after the person has said that he or she does not want to will likely engender animosity towards the client. C) Any employee of a telemarketing agency who tries to get someone to buy the product of a client of the agency after the person has said that he or she doesn't want to will engender animosity towards the client. D) Some people that an employee of a telemarketing agency calls to ask them to buy the product of a client of the agency will refuse to do so even though they are not predisposed to dislike the client. E) People who are already predisposed to dislike the client of a telemarketing agency are more likely to refuse to buy the product of that client than are people who are predisposed to like the client.

C) Any employee of a telemarketing agency who tries to get someone to buy the product of a client of the agency after the person has said that he or she doesn't want to will engender animosity towards the client. A and B: Discuss only the likelihood of the employee being able to determine this connection; we are looking for something to demonstrate that talking someone into buying a client's product will actually make that person dislike the client. D: doesn't address whether someone will end up disliking the client if asked to buy a product they refused to buy. E: focuses on a comparison between people who are already predisposed to dislike a client and the people predisposed to like them

Columnist: Taking a strong position on an issue makes one likely to misinterpret or ignore additional evidence that conflicts with one's stand. But in order to understand an issue fully, it is essential to consider such evidence impartially. Thus, it is best not to take a strong position on an issue unless one has already considered all important evidence conflicting with that position. The columnist's reasoning most closely conforms to which one of the following principles? (Practice test 5) A) It is reasonable to take a strong position on an issue if one fully understands the issue and has considered the evidence regarding that issue impartially B) To ensure that one has impartially considered the evidence regarding an issue on which one has taken a strong position, one should avoid misinterpreting or ignoring evidence regarding that issue C) Anyone who does not understand an issue fully should avoid taking a strong position on it D) One should try to understand an issue fully if doing so will help one to avoid misinterpreting or ignoring evidence regarding that issue E) It is reasonable to take a strong position on an issue only if there is important evidence conflicting with that position

C) Anyone who does not understand an issue fully should avoid taking a strong position on it A: This principle is backwards from what we see in the passage; the columnist tells us that understanding an issue fully is NECESSARY, not SUFFICIENT to take a strong position

Every delegate to the convention is a party member. Some delegates to the convention are government officials, and each government official who is at the convention is a speaker at the convention, as well. If the statements above are true, then which one of the following statements must be true? (Practice test 2) A) Every party member at the convention is a delegate at the convention? B) At least some speakers at the convention are neither delegates nor party members C) At least some speakers at the convention are delegates to the convention. D) All speakers at the convention are government officials. E) Every government official at the convention is a party member.

C) At least some speakers at the convention are delegates to the convention. If some delegates are gov officials, and every gov official is a speaker, we can infer that AT LEAST some of the delegates are speakers. A: Claim confuses a sufficient condition for a necessary one B: The stimulus never suggests that some speakers at the convention are neither delegates nor party members D: Confuses a sufficient condition for a necessary one. E: Contains an exaggeration; we know that some delegates are gov officials, and we can infer that AT LEAST SOME gov officials are party members, not that EVERY gov official is a party member

Practice Test 6 Economist: Owing to global economic forces since 1945, our country's economy is increasingly a service economy, in which manufacturing employs an ever smaller fraction of the workforce. Hence, we have engaged in less and less international trade. Which one of the following, if true, would most help to explain the decreasing engagement in international trade by the economist's country? A) International trade agreements have usually covered both trade in manufactured goods and trade in services. B) Employment in the service sector tends to require as many specialized skills as does employment in manufacturing. C) Because services are usually delivered in person, markets for services tend to be local. D) Many manufacturing jobs have been rendered obsolete by advances in factory automation. E) Some services can be procured less expensively from providers in other countries than from providers in the economist's country.

C) Because services are usually delivered in person, markets for services tend to be local. Explanation: Choice A makes the turn of events more mysterious, doesn't make sense. Choice B is irrelevant. Choice D gives a potential reason for why the country's economy is increasingly a service economy, but doesn't relate to the question at hand. Choice E deepens the mystery instead of resolving it; it the services in the economist's country can be obtained cheaper in other countries, the country would have some reason TO engage in international trade.

Had the party's economic theories been sound and had it succeeded in implementing its program, the inflation rate would have lessened considerably. But because the inflation rate actually increased, the party's economic theories were far off the mark. The flawed reasoning in which of the following arguments most closely resembles the flawed reasoning in the argument above? (practice test 1) A) If the people who inhabited the valley for so long had been invaded, or if there had been a dramatic climatic change, there would have been changes in the valley's architecture. But architecture in the valley remained the same throughout their stay. Thus, the valley people must not have been invaded at any time during their stay. B) Many people fear that if the opposition party wins the election and keeps its promise to cut wages dramatically, workers in key industries will strike. But because the workers have promised not to strike, these workers must think the party will not keep its promise of a dramatic wage cut. C) If the company had succeeded in selling its subsidiaries and used the cash to purchase the new patent, its stock price would have doubled in the last two years. But the price of the stock did not increase in that time. Thus, the company must have failed to sell its subsidiaries. D) City residents were expected to show a great deal of support for the rebels if the battle was won and the jailed rebel leaders freed. Residents have shown a great deal of support for the rebels for the last three days. Therefore, the rebels must have won the battle. E) If the TV station's new weather forecasting equipment had been worth the investment, the accuracy of its forecasts would have risen, along with its ratings. But the station's ratings actually decreased. Thus, the new equipment is no improvement on the old.

C) If the company had succeeded in selling its subsidiaries and used the cash to purchase the new patent, its stock price would have doubled in the last two years. But the price of the stock did not increase in that time. Thus, the company must have failed to sell its subsidiaries. Original argument structure: If X and Y, then Z. However, Z did not happen, so X must not have happened. This is the same flaw that's expressed in this answer choice.

A manager cannot extract the best performance from employees by threatening them with termination or offering financial rewards for high productivity. Rather, employees must come to want to do a good job for its own sake. One of the best ways for a manager to achieve this is to delegate responsibility to them, especially for decisions that previously had to be made by the manager. Which one of the following propositions is best illustrated by the situation described in the passage? (practice test 6) A) Increased responsibility can improve a person's sense of how power should be used. B) It is often the case that the desire for prestige is more powerful than the desire for job security. C) In some cases one's effectiveness in a particular role can be enhanced by a partial relinquishing of control. D) People who carry out decisions are in the best position to determine what those decisions should be. E) Business works best by harnessing the self-interest of individuals to benefit the company as a whole.

C) In some cases one's effectiveness in a particular role can be enhanced by a partial relinquishing of control. Manager can become more effective by delegating responsibility to an employee. A: Includes the idea of improving a person's sense of how power should be used, which isn't present in the passage. B: Passage never discusses either a desire for prestige or a desire for job security. D) Doesn't match passage situation, passage never addresses who is in the best position to determine decision made. E) Passage never discusses anything about benefiting the company as a whole.

Farmer: My neighbor claims that my pesticides are spreading to her farm in runoff water, but she is wrong. I use only organic pesticides, and there is no evidence that they harm either people or domestic animals. Furthermore, I am careful to avoid spraying on my neighbor's land. Which one of the following most accurately describes a reasoning flaw in the farmer's argument? (practice test 10) A) It treats a lack of evidence that organic pesticides harm people or domestic animals as proof that they cannot do so. B) It presumes, without providing justification, that being careful to avoid something usually results in its avoidance C) It does not address the neighbor's claim that pesticides used by the farmer are spreading onto her land. D) It fails to provide an alternative explanation for the presence of pesticides on the neighbor's land E) It ignores the possibility that pesticides might have dangerous effects other than harming people or domestic animals

C) It does not address the neighbor's claim that pesticides used by the farmer are spreading onto her land.

Biologist: Scientists have discovered fossilized bacteria in rocks 3.5 billion years old. The fossils indicate that these bacteria were quite complex and so must have already had a long evolutionary history when fossilized 3.5 billion years ago. However, Earth is only 4.6 billion years old, so the first life on Earth must have appeared soon after the planet's formation, when conditions were extremely harsh. This suggests that life may be able to arise under many difficult conditions throughout the universe. Which one of the following most accurately describes the role played in the biologist's argument by the claim that the fossilized bacteria discovered in rocks 3.5 billion years old must have had a long evolutionary history? (practice test 3) A) It is a claim for which no support is provided in the argument, and that is used to illustrate the conclusion of the argument as a whole B) It is a claim for which no support is provided in the argument, and that is used to support a claim that in turn lends support to the conclusion of the argument as a whole C) It is a claim for which some support is provided in the argument, and that itself is used to support another claim that in turn lends support to the conclusion of the argument as a whole D) It is a claim for which some support is provided in the argument, and that itself is not used to support any other claim in the argument E) It is a claim for which some support is provided in the argument, and that itself is used to support two distinct conclusions, neither of which is intended to provide support for the other

C) It is a claim for which some support is provided in the argument, and that itself is used to support another claim that in turn lends support to the conclusion of the argument as a whole This claim is supported by the claim that the fossils indicate that the bacteria were quite complex, and it is part of the support for the intermediate conclusion that life on earth must have appeared soon after the planet's formation

At a large elementary school researchers studied a small group of children who successfully completed an experimental program in which they learned to play chess. The study found that most of the children who completed the program soon showed a significant increase in achievement levels in all of their schoolwork. Thus, it is likely that the reasoning power and spatial intuition exercised in chess-playing also contribute to achievement in many other areas of intellectual activity. Which one of the following, if true, most tends to undermine the argument? (practice test 6) A) some students who did not participate in the chess program had learned to play chess at home. B) Those children who began the program but who did not successfully complete it had lower preprogram levels of achievement than did those who eventually did successfully complete the program. C) Many of the children who completed the program subsequently sought membership on a school chess team that required a high grade average for membership. D) Some students who did not participate in the chess program participated instead in after-school study sessions that helped them reach much higher levels of achievement in the year after they attended the session. E) At least some of the students who did not successfully complete the program were nevertheless more talented chess players than some of the students who did complete the program.

C) Many of the children who completed the program subsequently sought membership on a school chess team that required a high grade average for membership. This answer choice offers a different reason why the phenomenon occurred, which weakens the argument. A: Neither strengthens nor weakens argument because it focuses on students who did not participate in the chess program. B: Neither strengthens nor weakens argument. The preprogram levels of achievement don't impact anything. D: No impact on argument since the passage never claims that the experimental chess program is the only way to improve academic performance. E: Relative skill in chess doesn't have an impact on the argument, which is focused on higher school performance.

People who are allergic to cats are actually allergic to certain proteins found in the animals' skin secretions and saliva; which particular proteins are responsible, however, varies from allergy sufferer to allergy sufferer. Since all cats shed skin and spread saliva around their environment, there is no such thing as a cat incapable of provoking allergic reactions, although it is common for a given cat to cause an allergic reaction in some—but not all—people who are allergic to cats. Which one of the following statements is most strongly supported by the information above? (practice test 2) A) Any particular individual will be allergic to some breeds of cat but not to others. B) No cat is capable of causing an allergic reaction in all types of allergy sufferers. C) Not all cats are identical with respect to the proteins contained in their skin secretions and saliva D) The allergic reactions of some people who are allergic to cats are more intense than the allergic reactions of other allergy sufferers E) There is no way to predict whether a given cat will produce an allergic reaction in a particular allergy sufferer

C) Not all cats are identical with respect to the proteins contained in their skin secretions and saliva Can be inferred from the statement in the stimulus that not all people allergic to cats will have a reaction to a given cat A: We have no reason to believe that everyone is allergic to cats. B: The stimulus doesn't state that it's impossible for some cat to cause an allergic reaction in ALL types of allergy sufferers. D: Stimulus contains no information about the intensity of reactions. E: too extreme (NO way)

The more sunlight our planet reflects back into space, the cooler the global atmosphere tends to become. Snow and ice reflect much more sunlight back into space than do ocean water or land without snow cover. Therefore, the greater the area of Earth's surface that is covered with snow and ice, the cooler, on average, the global atmosphere is likely to become. Which one of the following, if true, would most strengthen the argument? (practice test 7) A) Low atmospheric temperatures are required for the formation of clouds that result in snow B) Other factors besides the reflectivity of ice and snow affect the cooling of Earth's atmosphere C) Ocean water and land heated by sunlight in turn warm the atmosphere D) The atmosphere derives most of its heat from the passage of sunlight through it E) Lighter-colored soil reflects more sunlight back into space than darker-colored soil

C) Ocean water and land heated by sunlight in turn warm the atmosphere Strengthens the argument by providing additional evidence in support of its conclusion. This taken with the information given in the stimulus makes the author's conclusion more supported. D: Conclusion is regarding what happens after the sunlight reaches the earth's surface, not what happens before

Traditional hatcheries raise fish in featureless environments and subject them to dull routines, whereas new, experimental hatcheries raise fish in visually stimulating environments with varied routines. When released into the wild, fish from the experimental hatcheries are bolder than those from traditional hatcheries in exploring new environments and trying new types of food. Fish raised in the experimental hatcheries, therefore, are more likely to survive after their release. Which one of the following is an assumption required by the argument? (Practice test 11) A) It is economically feasible for hatchery operators to expose fish to greater visual stimulation and to more varied routines B) The quality of the environments into which hatchery-raised fish are released has little effect on the fish's survival rate C) Some fish raised in traditional hatcheries die because they are too timid in their foraging for food D) Hatchery-raised fish that are released into the wild need to eat many different types of food to survive E) Fish in the wild always live in visually stimulating environments

C) Some fish raised in traditional hatcheries die because they are too timid in their foraging for food

Counselor: To be kind to someone, one must want that person to prosper. Yet, even two people who dislike each other may nevertheless treat each other with respect. And while no two people who dislike each other can be fully content in each other's presence, any two people who do not dislike each other will be kind to each other. If the counselor's statements are true, then which one of the following must be false? (practice test 11) A) Some people who like each other are not fully content in each other's presence B) Some people who are fully content in each other's presence do not want each other to prosper C) Some people who treat each other with respect are not fully content in each other's presence D) Some people who want each other to prosper dislike each other E) Some people who are kind to each other do not treat each other with respect

C) Some people who treat each other with respect are not fully content in each other's presence

The number of serious traffic accidents (accidents resulting in hospitalization or death) that occurred on Park Road from 1986 to 1990 was 35 percent lower than the number of serious accidents from 1981 to 1985. The speed limit on Park Road was lowered in 1986. Hence, the reduction of the speed limit led to the decrease in serious accidents. Which one of the following, if true, most weakens the argument? (Practice test 5) A) The number of speeding tickets issued annually on Park Road remained roughly constant from 1981 to 1990. B) Beginning in 1986, police patrolled Park Road much less frequently than in 1985 and previous years. C) The annual number of vehicles using Park Road decreased significantly and steadily from 1981 to 1990 D) The annual number of accidents on Park Road that did not result in hospitalization remained roughly constant from 1981 to 1990 E) Until 1986 accidents were classified as "serious" only if they resulted in an extended hospital stay

C) The annual number of vehicles using Park Road decreased significantly and steadily from 1981 to 1990 Weakens the argument by providing a DIFFERENT reason for the decrease in serious accidents A: neither strengthens nor weakens the argument since we can't definitively connect the number of speeding tickets issued to the number of serious accidents.

Only engineering is capable of analyzing the nature of a machine in terms of the successful working of the whole; physics and chemistry determine the material conditions necessary for this success, but cannot express the notion of purpose. Similarly, only physiology can analyze the nature of an organism in terms of organs' roles in the body's healthy functioning. Physics and chemistry cannot ascertain by themselves any of these operational principles. Which one of the following is an assumption required by the analogy? (practice test 7) A) The functioning of the human organism is machine-like in nature. B) Physics and chemistry determine the material conditions required for good physiological functioning. C) The notion of purpose used by engineers to judge the success of machinery has an analog in organisms. D) Physiology as a science is largely independent of physics and chemistry E) Biological processes are irreducible to mechanical or chemical processes.

C) The notion of purpose used by engineers to judge the success of machinery has an analog in organisms. If this choice weren't true, the analogy wouldn't make any sense. A: Unnecessary; argument just needs physiology's role with respect to organisms to be similar in one particular way to engineering's role with respect to machines

Many bird and reptile species use hissing as a threat device against potential predators. The way these species produce hissing sounds is similar enough that it is likely that this behavior developed in an early common ancestor. At the time this common ancestor would have lived, however, none of its potential predators would have yet acquired the anatomy necessary to hear hissing sounds. Which of the following, if true, most helps to resolve the apparent discrepancy in the information above? (practice test 6) A) Like its potential predators, the common ancestor of bird and reptile species would have lacked the anatomy necessary to hear hissing sounds. B) The common ancestor or bird and reptile species would probably have employed multiple threat devices against potential predators. C) The production of a hissing sound would have increased the apparent body size of the common ancestor of bird and reptile species. D) The use of hissing as a threat device would have been less energetically costly than other threat behaviors available to the common ancestor of bird and reptile species. E) Unlike most modern bird and reptile species, the common ancestor of these species would have had few predators.

C) The production of a hissing sound would have increased the apparent body size of the common ancestor of bird and reptile species. The discrepancy: how can both birds and reptiles use hissing when at the time that it existed, none of its predators could have heard hissing sounds? A: Doesn't help us understand how birds and reptiles could have evolved to use hissing. B: Whether the common ancestor used just one or multiple threat devices doesn't help explain how hissing could have evolved. D: Doesn't explain why hissing was ever used as a threat device if predators couldn't hear it E: Whether the ancestor had few predators or many, we still need to understand why if would have used hissing as a threat if its predators couldn't hear it.

Normally, political candidates send out campaign material in order to influence popular opinion. But the recent ads for Ebsen's campaign were sent to too few households to serve this purpose effectively. The ads were evidently sent out to test their potential to influence popular opinion. They covered a wide variety of topics, and Ebsen's campaign has been spending heavily on follow-up to gauge their effect on recipients. Which one of the following most accurately expresses the conclusion drawn in the argument above? (practice test 11) A) Normally, political candidates send out campaign material to influence popular opinion B) The recent ads for Ebsen's campaign were sent to too few households to influence popular opinion effectively. C) The recent ads for Ebsen's campaign were sent out to test their potential to influence popular opinion D) The recent ads for Ebsen's campaign covered a wide variety of topics E) Ebsen's campaign has been spending heavily on follow-up surveys to gauge the ads' effect on recipients

C) The recent ads for Ebsen's campaign were sent out to test their potential to influence popular opinion

Australia has considerably fewer species of carnivorous mammals than any other continent does but about as many carnivorous reptile species as other continents do. This is probably a consequence of the unusual sparseness of Australia's ecosystems. To survive, carnivorous mammals must eat much more than carnivorous reptiles need to; thus carnivorous mammals are at a disadvantage in ecosystems in which there is relatively little food. Which one of the following most accurately expresses the main conclusion of the argument? (practice test 5) A) Australia has considerably fewer species of carnivorous mammals than any other continent does but about as many carnivorous reptile species as other continents do. B) In ecosystems in which there is relatively little food, carnivorous mammals are at a disadvantage relative to carnivorous reptiles. C) The unusual sparseness of Australia's ecosystems is probably the reason Australia has considerably fewer carnivorous mammal species than other continents do but about as many carnivorous reptile species D) The reason that carnivorous mammals are at a disadvantage in ecosystems in which there is relatively little food is that they must eat much more in order to survive than carnivorous reptiles need to E) Because Australia's ecosystems are unusually sparse, carnivorous mammals there are at a disadvantage relative to carnivorous reptiles.

C) The unusual sparseness of Australia's ecosystems is probably the reason Australia has considerably fewer carnivorous mammal species than other continents do but about as many carnivorous reptile species Arguer attempts to prove this statement by using other statements in the passage as support

Patricia: During Japan's Tokugawa period, martial arts experts known as ninjas were trained for the purposes of espionage and assassination. Yet at that time there was actually very little ninja activity in Japan, and most Japanese did not fear ninjas. Tamara: That is not true. Many wealthy Japanese during the Tokugawa period had their houses constructed with intentionally squeaky floors so that they would receive warning if a ninja were in the house. Of the following, which one, if true, is the strongest counter Patricia can make to Tamara's objection? (practice test 1) A) Many poor Japanese during the Tokugawa period also had houses constructed with intentionally squeaky floors. B) As part of their secret training, ninjas learned to walk on squeaky floors without making a sound. C) The wealthy made up a small portion of Japan's population during the Tokugawa period. D) The fighting prowess of ninjas was exaggerated to mythic proportions in the years following the Tokugawa period. E) There were very few ninjas at any time other than during the Tokugawa period.

C) The wealthy made up a small portion of Japan's population during the Tokugawa period. Strengthens argument by indicating that the wealthy Japanese (cited by Tamara) only made up a small portion of the population. A: helps Tamara's argument by adding an even larger group of people who seemed to fear ninjas. B: Doesn't actually impact Tamara's claim that people feared ninjas D: The fighting proficiency of ninjas neither strengthens nor weakens the argument E: Irrelevant

To get the free dessert, one must order an entree and a salad. But anyone who orders either an entree or a salad can receive a free soft drink. Thus, anyone who is not eligible for a free soft drink is not eligible for a free dessert. The reasoning in the argument above is most similar to the reasoning in which one of the following arguments? (Practice test 5) A) To get an executive position at Teltech, one needs a university diploma and sales experience. But anyone who has worked at Teltech for more than six months who does not have sales experience has a university diploma. Thus, one cannot get an executive position at Teltech unless one has worked there for six months. B) To be elected class president, one must be well liked and well known. Anyone who is well liked or well known has something better to do than run for class president. Therefore, no one who has something better to do will be elected class president. C) To grow good azaleas, one needs soil that is both rich in humus and low in acidity. Anyone who has soil that is rich in humus or low in acidity can grow blueberries. So, anyone who cannot grow blueberries cannot grow good azaleas. D) To drive to Weller, one must take the highway or take Old Mill Road. Anyone who drives to Weller on the highway will miss the beautiful scenery. Thus, one cannot see the beautiful scenery without taking Old Mill Road to Weller. E) To get a discount on ice cream, one must buy frozen raspberries and ice cream together. Anyone who buys ice cream or raspberries will get a coupon for a later purchase. So, anyone who does not get the discount on ice cream will not get a coupon for a later purchase.

C) To grow good azaleas, one needs soil that is both rich in humus and low in acidity. Anyone who has soil that is rich in humus or low in acidity can grow blueberries. So, anyone who cannot grow blueberries cannot grow good azaleas. Structure: To X, need Y and Z. If Y, then Q. So, if NOT Q, then NOT X

Some heartburn-medication advertisements imply that unrelieved heartburn is likely to cause esophageal cancer. This is simply false. The fact is that only about 5 percent of people with severe heartburn have a condition called Barrett's esophagus, in which cells similar to those in the stomach's lining develop in the lower esophagus. Only these people have an increased risk of developing cancer because of heartburn. Which one of the following most accurately expresses the overall conclusion drawn in the argument? (practice test 3) A) Only those people with Barrett's esophagus can suffer an increased risk of developing cancer from heartburn B) An increase in the risk of esophageal cancer arises from cell similar to those in the stomach's lining developing in the lower esophagus C) Unrelieved heartburn is not likely to cause esophageal cancer D) Some heartburn medication advertisements imply that unrelieved heartburn is likely to cause esophageal cancer E) The dangers touted by heartburn medication advertisements will affect relatively few of the people that see those advertisements

C) Unrelieved heartburn is not likely to cause esophageal cancer "This is simply false" is saying that unrelieved heartburn is not likely to cause esophageal cancer E: This is a paraphrase of the argument's support but is not the overall conclusion

Ethicist: Only when we know a lot about the events that led to an action are we justified in praising or blaming a person for that action—as we sometimes are. We must therefore reject Tolstoy's rash claim that if we knew a lot about the events leading up to any action, we would cease to regard that action as freely performed. Which one of the following, if assumed, enables the conclusion of the ethicist's argument to be properly drawn? (practice test 2) A) People should not be regarded as subject to praise or blame for actions that were caused by conditions beyond their control. B) Whether an act is one for which the person doing it is genuinely responsible is not determined by how much information others possess about that act. C) We can be justified in praising or blaming a person for an action only when we regard that action as freely performed D) The responsibility a person bears for an action is not a matter of degree; however, our inclination to blame or praise whoever performed the action varies with the amount of information available. E) If we do not know much about the events leading up to any given action, we will regard that action as freely performed.

C) We can be justified in praising or blaming a person for an action only when we regard that action as freely performed If both conditions given by the stimulus are satisfied at the same time, we can know a lot about the events that led to an action and still regard it as freely performed. A: The concept of an action being caused by conditions beyond one's control is not analogous to actions not being freely performed B: Fails to establish missing link between the premise and the conclusion D: Introduces an entirely unrelated concept into the argument E: This choice merely confuses the sufficient condition in his claim for a necessary one.

Evolution does not always optimize survival of an organism. Male moose evolved giant antlers as a way of fighting other males for mates, giving those with the largest antlers an evolutionary advantage. But those antlers also make it harder to escape predators, since they can easily get tangled in trees. All male moose would be better off with antlers half the current size: they would all be less vulnerable to predators, and those with the largest antlers would maintain their relative advantage. Which one of the following is a technique of reasoning used in the argument? (practice test 8) A) citing an example to cast doubt on a competing argument B) employing an analogy in order to dispute a generalization C) challenging a general claim by presenting a counterexample D) disputing the relevance of an example thought to support an opposing view E) undermining a claim by showing that it is self-contradictory

C) challenging a general claim by presenting a counterexample

Politician: Union leaders argue that increases in multinational control of manufacturing have shifted labor to nations without strong worker protections, resulting in a corresponding global decrease in workers' average wages. Given that these leaders have a vested interest in seeing wages remain high, they would naturally want to convince legislators to oppose multinational control. Thus, legislators should reject this argument. The reasoning in the politician's argument is flawed in that the argument (Practice test 6) A) treats the mere fact that certain people are union members as sufficient to cast doubt on all the viewpoints expressed by those people B) presumes, without providing justification, that anyone whose political motivations are clearly discernible is an unreliable source of information to legislators C) treats circumstances potentially affecting the union leaders' argument as sufficient to discredit those leaders' argument D) presumes, without providing justification, that the argument it cites is the union leaders' only argument for their view E) presumes, without providing evidence, that leaders of all unions argue against increases in multinational control of manufacturing

C) treats circumstances potentially affecting the union leaders' argument as sufficient to discredit those leaders' argument The flaw in the argument is that just because someone has a vested interest in a stance they take doesn't automatically mean that their stance is wrong. Choice A: too strong - politician doesn't discount ALL viewpoints Choice B: irrelevant because political motivations are never mentioned in the passage Choice D: we have no reason to believe that the politician believes the union leaders only have one argument Choice D: too strong- the politician doesn't make a statement about leaders of ALL unions.

Alissa: If, as the mayor says, the city can no longer continue to fund both the children's museum and local children's television programming, then it should cease funding the television programming. The interactive character of the exhibits at the museum makes for a richer educational experience than watching television, which is largely passive. Greta: We should stop funding the museum, not the television programming, because, as the mayor has also pointed out, the museum reaches a much smaller audience. On the basis of their statements, it can be inferred that Alissa and Greta disagree on which one of the following? A) whether the city will need to cease funding local children's TV programming if it continues funding the children's museum B) whether the mayor has spoken truthfully about what will need to happen if the city does not cease funding local children's TV programs C) whether the city should cease funding local children's TV programming if continuing to fund it would mean that the city would have to cease funding the children's museum D) whether local children's TV programming provides a beneficial educational experience to a greater number of children in the city than does the children's museum E) whether the children's museum provides a rich experience for those children who visit it

C) whether the city should cease funding local children's TV programming if continuing to fund it would mean that the city would have to cease funding the children's museum Correct because they disagree over which project should receive the funding A: Both agree that both things cannot be funded B: Neither of them question the mayor's truthfulness D: Alissa voices no opinion on this E: Greta does not give an opinion on this

Editor: The city's previous recycling program, which featured pickup of recyclables every other week, was too costly. The city claims that its new program, which features weekly pickup, will be more cost effective, since the greater the volume of recyclables collected per year, the more revenue the city gains from selling the recyclables. But this is absurd. People will put out the same volume of recyclables overall; it will just be spread out over a greater number of pickups. Which one of the following, if true, most weakens the editor's argument? (Practice test 6) A) The cost of collecting and disposing of general trash has been less than the cost of collecting and disposing of recyclables, and this is still likely to be the case under the new recycling program B) Even if the volume of collected recyclables increases, that increase might not be enough to make the recycling program cost effective. C) Because the volume of recyclables people accumulate during a week is less than what they accumulate during two weeks, the city expects a recyclables pickup to take less time under the new program. D) A weekly schedule for recyclables pickup is substantially easier for people to follow and adhere to than a schedule of pickups every other week. E) Because of the increase in the number of pickups under the new program, the amount charged by the contractor that collects the city's recyclables will increase significantly.

D) A weekly schedule for recyclables pickup is substantially easier for people to follow and adhere to than a schedule of pickups every other week. If it's true that people will follow a weekly schedule better than the previous schedule, we are inclined to believe that people will put out more recyclables overall. A: No impact on the argument since it compares the general trash costs with the recycling costs. B:Closer to strengthening argument than weakening it. C: No impact; the passage doesn't refer to the time spent on pickups, so this is irrelevant. E: May actually strengthen argument in that the new program may not be more cost-effective

The airport's runways are too close to each other to allow simultaneous use of adjacent runways when visibility is poor, so the airport allows only 30 planes an hour to land in poor weather; in good weather 60 planes an hour are allowed to land. Because airline schedules assume good weather, bad weather creates serious delays. Which one of the following is most strongly supported by the information above? (practice test 1) A) In poor weather, only half as many planes are allowed to land each hour on any one runway at the airport as are allowed to land on it in good weather. B) When the weather at the airport is good, it is likely that there are planes landing on two adjacent runways at any given time. C) If any two of the airport's runways are used simultaneously, serious delays result. D) Airlines using the airport base their schedules on the assumption that more than 30 planes an hour will be allowed to land at the airport. E) In good weather, there are few if any seriously delayed flights at the airport.

D) Airlines using the airport base their schedules on the assumption that more than 30 planes an hour will be allowed to land at the airport. Combining the statements provided in the stimulus, we can infer that airlines assume a scenario in which more than 30 planes an hour are allowed to land at the airport. A: We cannot infer that only half as many planes are allowed to land each hour on any one runway. B:Don't add information that's not in the stimulus; it never states that when the weather is good, it is likely that adjacent runways are being used. We only know the likelihood of that in bad weather. D: Connection between simultaneous use of runways and serious delays is never mentioned in the passage E: We are never given any info that would allow us to infer how many delays happen in good weather.

It is likely that most of the new television programs Wilke & Wilke produce for this season will be canceled. Most of the new shows they produced last season were canceled due to insufficient viewership. Furthermore, their new shows are all police dramas, and few police dramas have been popular in recent years. Which of the following, if true, most helps to strengthen the argument? A) Wilke & Wilke have produced more new shows for this season than they produced last season. B) Most of the shows that Wilke & Wilke produced last year were police dramas. C) None of the shows that Wilke & Wilke produced last year that were not canceled were police dramas. D) All of the new shows that Wilke & Wilke produced last year that were canceled were police dramas. E) None of the most popular TV shows last year were police dramas.

D) All of the new shows that Wilke & Wilke produced last year that were canceled were police dramas. This choice would indicate that Wilke & Wilke are continuing a trend that didn't work out for them last year. A: Neither strengthens nor weakens. B: If MOST of Wilke & Wilke's shows from last season were canceled and MOST of their shows were police dramas, we can only deduce that SOME of their canceled shows from last season were police dramas. This isn't enough to draw the conclusion that MOST new shows will be canceled. C: Doesn't strengthen the argument because it leaves the possibility that Wilke and Wilke didn't produce any police dramas last year. May weaken the argument. E: Neither strengthens nor weakens argument, just confirms what was provided in passage.

Any literary translation is a compromise between two goals that cannot be entirely reconciled: faithfulness to the meaning of the text and faithfulness to the original author's style. Thus, even the most skillful translation will be at best a flawed approximation of the original work. Which one of the following principles, if valid, most helps to justify the reasoning in the argument above? (practice test 7) A) A translation of a literary work should be entirely faithful to neither the meaning of the text nor the original author's style B) If a literary translation is flawed as an approximation of the original work, it cannot be regarded as a successful compromise between faithfulness to the meaning of the text and faithfulness to the original author's style C) The most skillful literary translation of a work will not necessarily be the most balanced compromise between faithfulness to the meaning of the text and faithfulness to the original author's style D) Any translation that is not entirely faithful to both the meaning of the text and the original author's style will be at best a flawed approximation of that work E) Not even the most skillful literary translation could be faithful to both the literal meaning of the text and the original author's style.

D) Any translation that is not entirely faithful to both the meaning of the text and the original author's style will be at best a flawed approximation of that work This principle offers a clear conditional relationship, which is reflected in the argument given E: Restates something that can already be inferred from the given support; doesn't help to bridge the gap to the conclusion that even the most skillful literary translations are AT BEST FLAWED APPROXIMATIONS OF THE ORIGINAL

The level of triglycerides in the blood rises when triglycerides are inadequately metabolized. Research shows that patients with blood triglyceride levels above 1 milligram per milliliter are twice as prone to heart attacks as others. Thus, it is likely that consuming large amounts of fat, processed sugar, or alcohol, each known to increase triglyceride levels in the blood, is a factor causing heart disease. Which one of the following, if true, most weakens the argument? (practice test 11) A) People with a high-fat diet who engage in regular, vigorous physical activity are much less likely to develop heart disease than are sedentary people with a low-fat diet B) Triglyceride levels above 2 milligrams/milliliter increase the risk of some serious illnesses not related to heart disease C) Shortly after a person ceases to regularly consume alcohol and processed sugar, that person's triglyceride levels drop dramatically D) Heart disease interferes with the body's ability to metabolize triglycerides E) People who maintain strict regimens for their health tend to adopt low-fat diets and to avoid alcohol and processed sugar

D) Heart disease interferes with the body's ability to metabolize triglycerides

Because no other theory has been able to predict it so simply and accurately, the advance of the perihelion of Mercury is sometimes cited as evidence in support of Einstein's theory of general relativity. However, this phenomenon was already well known when Einstein developed his theory, and he quite probably adjusted his equations to generate the correct numbers for the perihelion advance. Therefore, accounting for this advance should not be counted as evidence in support of Einstein's theory. Which one of the following principles, if valid, most helps to justify the argument above? (practice test 4) A) Unless a phenomenon predicted by a scientific theory is unknown at the time the theory is developed, the theory should not be credited with the discovery of that phenomenon B) A phenomenon that is predicted by a scientific theory should not count as evidence in favor of that theory unless the theory was developed with that phenomenon in mind C) Unless a theory can accurately account for all relevant phenomena that are already well known at the time of its development, it cannot be regarded as well supported D) If a theory is adjusted specifically to account for some particular phenomenon, a match between that theory and that phenomenon should not count as evidence in favor of the theory E) If a theory is adjusted to generate the correct predictions for some phenomenon that is already known to the scientist developing the theory, the theory should not be counted as predicting that phenomenon.

D) If a theory is adjusted specifically to account for some particular phenomenon, a match between that theory and that phenomenon should not count as evidence in favor of the theory E: the author argues that the phenomenon shouldn't be counted as evidence SUPPORTING THE THEORY, not that the theory shouldn't be counted as PREDICTING THE PHENOMENON

In a recent study, one group of participants watched video recordings of themselves running on treadmills, and a second group watched recordings of other people running on treadmills. When contacted later, participants in the first group reported exercising, on average, 1 hour longer each day than did the other participants. This shows that watching a recording of yourself exercising can motivate you to exercise more. Which one of the following, if true, most weakens the argument? (practice test 8) A) In another study, people who watched recordings of themselves lifting weights exercised for more time each day than did people who watched recordings of themselves running B) Another study's members exhibited an increased willingness to give to charity after hearing stories in which people with whom they identified did so C) Participants who were already highly motivated to exercise did not report exercising for any longer each day than they have before the study D) In studies of identical twins, participants who observed their twin reading overreported by a significant amount how much time they themselves spent reading in the days that followed E) A third group of participants who watched recordings of themselves sitting on couches afterwards reported being sedentary for more time each day than did the other participants

D) In studies of identical twins, participants who observed their twin reading overreported by a significant amount how much time they themselves spent reading in the days that followed If people who watch themselves doing something tend to overreport the number of hours they spend actually doing that thing, the argument would be weakened

Editorial: Cell-phone usage on buses and trains is annoying to other passengers. This suggests that recent proposals to allow use of cell phones on airplanes are ill-advised. Cell-phone use would be far more upsetting on airplanes than it is on buses and trains. Airline passengers are usually packed in tightly. And if airline passengers are offended by the cell-phone excesses of their seatmates, they often cannot move to another seat. Which one of the following most accurately describes the role played in the editorial's argument by the statement that cell-phone use would be far more upsetting on airplanes than it is on buses and trains? (practice test 9) A) It is the main conclusion of the argument B) It is a claim that the argument tries to rebut C) It is a premise that indirectly supports the main conclusion of the argument by supporting a premise for that conclusion D) It is a conclusion for which support is provided and that itself is used in turn to directly support the argument's main conclusion E) It provides background information that plays no role in the reasoning in the argument

D) It is a conclusion for which support is provided and that itself is used in turn to directly support the argument's main conclusion statement in question DIRECTLY supports the main conclusion that cell phone use shouldn't be allowed on airplanes. Statement is supported by the premises explaining why cell phone use would be more annoying

Ethicist: The penalties for drunk driving are far more severe when the drunk driver accidentally injures people than when no one is injured. Moral responsibility for an action depends solely on the intentions underlying the action and not on the action's results. Therefore, legal responsibility, depending as it does in at least some cases on factors other than the agent's intentions, is different than moral responsibility. The claim that the penalties for drunk driving are far more severe when the drunk driver accidentally injures people than when no one is injured plays which one of the following roles in the ethicist's argument? (Practice Test 5) A) It is a premise offered in support of the claim that legal responsibility for an action is based solely upon features of the action that are generally unintended by the agent B) It is offered as an illustration of the claim that the criteria of legal responsibility for an action include but are not the same as those for moral responsibility C) It is offered as an illustration of the claim that people may be held morally responsible for an action for which they are not legally responsible D) It is a premise offered in support of the claim that legal responsibility depends in at least some cases on factors other than the agent's intentions E) It is a premise offered in support of the claim that moral responsibility depends solely on the intentions underlying the action and not on the action's result

D) It is a premise offered in support of the claim that legal responsibility depends in at least some cases on factors other than the agent's intentions Statement in question is provided as support for the intermediate conclusion that legal responsibility depends in at least some cases on factors other than the agent's intentions, which supports the overall conclusion

The position that punishment should be proportional to how serious the offense is but that repeat offenders should receive harsher punishments than first-time offenders is unsustainable. It implies that considerations as remote as what an offender did years ago are relevant to the seriousness of an offense. If such remote considerations were relevant, almost every other consideration would be too. But this would make determining the seriousness of an offense so difficult that it would be impossible to apply the proportionality principle. The statement that considerations as remote as what an offender did years ago are relevant to the seriousness of an offense plays which one of the following roles in the argument? (practice test 8) A) It is a statement the argument provides grounds to accept and from which the overall conclusion is inferred B) It is a statement inferred from a position the argument seeks to defend C) It is the overall conclusion in favor of which the argument offers evidence D) It is an allegedly untenable consequence of a view rejected in the argument's overall conclusion E) It is a premise offered in support of an intermediate conclusion of the argument

D) It is an allegedly untenable consequence of a view rejected in the argument's overall conclusion

Censor: All anarchist novels have two objectionable characteristics: a subversive outlook and the depiction of wholesale violence. Therefore, it is permissible to ban any anarchist novel that would do more harm than good to society. Which one of the following principles, if valid, most helps to justify the censor's reasoning? (practice test 10) A) If a novel has a subversive outlook but does not depict wholesale violence, it is impermissible to ban it B) If a novel depicts wholesale violence, then it is permissible to ban it if doing so would do more good than harm to society C) It is permissible to ban a novel only if the novel has a subversive outlook and would do more harm than good to society D) It is permissible to ban a novel that would cause society more harm than good if the novel has two or more objectionable characteristics E) It is permissible to ban a novel that depicts wholesale violence only if that novel has at least one other objectionable characteristic

D) It is permissible to ban a novel that would cause society more harm than good if the novel has two or more objectionable characteristics

Ethicist: Although science is frequently said to be morally neutral, it has a traditional value system of its own. For example, scientists sometimes foresee that a line of theoretical research they are pursuing will yield applications that could seriously harm people, animals, or the environment. Yet, according to science's traditional value system, such consequences do not have to be considered in deciding whether to pursue that research. Ordinary morality, in contrast, requires that we take the foreseeable consequences of our actions into account whenever we are deciding what to do. The ethicist's statements, if true, most strongly support which one of the following? (practice test 5) A) Scientists should not be held responsible for the consequences of their research B) According to the dictates of ordinary mortality, scientists doing research that ultimately turns out to yield harmful applications are acting immorally C) Science is morally neutral because it assigns no value to the consequences of theoretical research D) It is possible for scientists to both adhere to the traditional values of their field and violate a principle of ordinary morality E) The uses and effects of scientifically acquired knowledge can never be adequately foreseen

D) It is possible for scientists to both adhere to the traditional values of their field and violate a principle of ordinary morality According to the stimulus, when scientists don't take research consequences into account their actions both conform to the traditional values of science AND violate ordinary morality B: Not supported by passage. Passage only tells us that according to ordinary morality, scientists need to take foreseeable consequences under consideration when making decisions

Meteorologist: Heavy downpours are likely to become more frequent if Earth's atmosphere becomes significantly warmer. A warm atmosphere heats the oceans, leading to faster evaporation, and the resulting water vapor forms rain clouds more quickly. A warmer atmosphere also holds more moisture, resulting in larger clouds. In general, as water vapor in larger clouds condenses, heavier downpours are more likely to result. Which one of the following most accurately describes the role played in the meteorologist's argument by the claim that, in general, as water vapor in larger clouds condenses, heavier downpours are more likely to result? (practice test 3) A) It is the only conclusion in the argument B) It is the conclusion of the argument as a whole but is not the only explicitly stated conclusion in the argument C) It is a statement that the argument is intended to support, but is not the conclusion of the argument as a whole D) It is used to support the only conclusion in the argument. E) It provides a causal explanation of the phenomenon described by the conclusion of the argument as a whole, but it is not intended to provide support for that conclusion

D) It is used to support the only conclusion in the argument. Only conclusion is that heavy downpours are likely to increase if the atmosphere becomes warmer C: No other statement supports this claim, so it can't be the case that the argument is intending to support it

Editorialist: News media rarely cover local politics thoroughly, and local political business is usually conducted secretively. These factors each tend to isolate local politicians from their electorates. This has the effect of reducing the chance that any particular act of resident participation will elicit a positive official response, which in turn discourages resident participation in local politics. Which one of the following is most strongly supported by the editorialist's statements? (LSATMax Practice test) A) Particular acts of resident participation would be likely to elicit a positive response from local politicians if those politicians were less isolated from their electorate. B) Local political business should be conducted less secretively because this would avoid discouraging resident participation in local politics. C) The most important factor influencing a resident's decision as to whether to participate in local politics is the chance that the participation will elicit a positive official response. D) More-frequent thorough coverage of local politics would reduce at least one source of discouragement from resident participation in local politics. E) If resident participation in local politics were not discouraged, this would cause local politicians to be less isolated from their electorate.

D) More-frequent thorough coverage of local politics would reduce at least one source of discouragement from resident participation in local politics. Must be true questions are all about inferences, which are all about overlapping ideas. Linking together the chain of causality made it a lot easier to analyze the answer choices and we could identify that the situation was causal instead of conditional

Nutritionist: Most fad diets prescribe a single narrow range of nutrients for everyone. But because different foods contain nutrients that are helpful for treating or preventing different health problems, dietary needs vary widely from person to person. However, everyone should eat plenty of fruits and vegetables, which protect against a wide range of health problems. Which one of the following is most strongly supported by the nutritionist's statements? (practice test 8) A) Most fad diets require that everyone following them eat plenty of fruits and vegetables. B) Fruits and vegetables are the only foods that contain enough different nutrients to protect against a wide range of health problems. C) Any two people have different health problems and thus different dietary needs D) Most fad diets fail to satisfy the dietary needs of some people E) There are very few if any nutrients that are contained in every food other than fruits and vegetables

D) Most fad diets fail to satisfy the dietary needs of some people Given by the statement that MOST fad diets prescribe a single narrow range of nutrients for everyone but that dietary needs vary widely from person to person

Albert: The government has proposed new automobile emissions regulations designed to decrease the amount of polycyclic aromatic hydrocarbons (PAHs) released into the atmosphere by automobile exhaust. I don't see the need for such regulations; although PAHs are suspected of causing cancer, a causal link has never been proven. Erin: Scientists also blame PAHs for 10,000 premature deaths in this country each year from lung and heart disease. So the proposed regulations would save thousands of lives. Which one of the following, if true, is the logically strongest counter that Albert can make to Erin's argument? (Practice test 5) A) Most automobile manufacturers are strongly opposed to additional automobile emissions regulations B) It is not known whether PAHs are a causal factor in any diseases other than heart and lung disease and cancer C) Even if no new automobile emissions regulations are enacted, the amount of PAHs released into the atmosphere will decrease if automobile usage declines D) Most of the PAHs released into the atmosphere are the result of wear and tear on automobile tires E) PAHs are one of several components of automobile exhaust that scientists suspect of causing cancer

D) Most of the PAHs released into the atmosphere are the result of wear and tear on automobile tires Weakens Erin's argument because if most PAHs are released into the atmosphere due to TIRES, then the proposed regulations wouldn't do much to save thousands of lives. C: While it might be good news if PAHs could decrease without regulations, it's still possible that the regulations will cause the amount of PAHs released to decrease even MORE

Ethicist: Marital vows often contain the promise to love "until death do us part." If "love" here refers to a feeling, then this promise makes no sense, for feelings are not within one's control, and a promise to do something not within one's control makes no sense. Thus, no one—including those making marital vows—should take "love" in this context to be referring to feelings. The ethicist's conclusion follows logically if which one of the following is assumed? (Practice test 9) A) None of our feelings are within our control B) People should not make promises to do something that is not within their control C) "Love" can legitimately be taken to refer to something other than feelings D) Promises should not be interpreted in such a way that they make no sense E) Promises that cannot be kept do not make any sense

D) Promises should not be interpreted in such a way that they make no sense We are never told of any reason that you SHOULDN'T interpret a promise in a certain way if that promise didn't make sense; the ethicist ASSUMES that you shouldn't, and this choice confirms that assumption E: the ethicist doesn't address promises that CANNOT BE KEPT, but rather promises that are beyond our control

A film makes a profit if the number of people who see it is sufficient to generate revenues from ticket sales greater than the amount spent to make it. Hence, the primary goal of movie executives is to maximize the number of people who see a film. However, it is not the primary goal of television executives to maximize the number of viewers for their shows. Which one of the following, if true, most helps to explain the difference between the goals of movie executives and those of TV executives? (practice test 2) A) More people are willing to see a film more than once than are willing to watch a TV show more than once B) There is no analog in TV to the large profits that owners of movie theaters make by selling refreshments to their customers. C) The average cost of producing an hour of film is much greater than the average cost of producing an hour of TV D) TV shows make their profits from sponsors, who are chiefly concerned with the purchasing power of the people who watch the TV show E) Over half of the most popular TV shows are shows that viewers do not have to pay to watch.

D) TV shows make their profits from sponsors, who are chiefly concerned with the purchasing power of the people who watch the TV show If sponsors of TV shows are mostly concerned with purchasing power of those who watch shows, we would expect TV executives to share the sponsors' concerns. A: We still don't know why TV execs aren't concerned about the number of viewers they get B: Doesn't help us understand the difference in priorities between movie and TV execs. C: Doesn't explain the issue E: The fact that TV shows are free to watch doesn't explain why maximizing viewership isn't a priority.

If the city starts requiring residents to sort the materials that they put out for recycling, then many residents will put more recyclables in with their regular garbage. This will result in more recyclables being buried in the city's landfill. However, because of the cost of having city workers do the sorting, the sanitation department will not stay within its budget unless the sorting requirement for residents is implemented. Which of the following statements logically follows from the information above? (practice test 1) A) Most of the city's residents will continue to recycle even if a sorting requirement is implemented. B) If the city starts requiring residents to sort their recyclables, then all of the residents who continue to recycle will sort their recyclables C) Implementing the sorting requirement would not cause the city's annual cost of sending garbage to its landfill to exceed its current annual cost of sorting recyclable D) The amount of recyclables going to the city's landfill will increase if the sanitation department stays within its budget. E) If the city implements the sorting requirement, the sanitation department will stay within its budget

D) The amount of recyclables going to the city's landfill will increase if the sanitation department stays within its budget. A: We have no idea from the passage what percentage of the city's residents will continue recycling if the sorting requirement is implemented. B: We can infer that some residents will NOT sort their recyclables from the passage C: We don't have enough information to support this choice E: Confuses necessary and sufficient conditions

Political candidates' speeches are loaded with promises and with expressions of good intention, but one must not forget that the politicians' purpose in giving these speeches is to get themselves elected. Clearly, then, these speeches are selfishly motivated and the promises made in them are unreliable. Which one of the following most accurately describes a flaw in the argument above? (LSAT Max Practice test) A) The argument presumes, without providing justification, that if a person's promise is not selfishly motivated then that promise is reliable. B) The argument presumes, without providing justification, that promises made for selfish reasons are never kept. C) The argument confuses the effect of an action with its cause. D) The argument overlooks the fact that a promise need not be unreliable just because the person who made it had an ulterior motive for doing so. E) The argument overlooks the fact that a candidate who makes promises for selfish reasons may nonetheless be worthy of the office for which he or she is running.

D) The argument overlooks the fact that a promise need not be unreliable just because the person who made it had an ulterior motive for doing so. Noticing motives is important on any LSAT question. When a motive behind an action is stated, it's important to pay attention to it because it's frequently tested. Often arguments are flawed in the way that the reasons behind an action don't determine the outcome of that action

Professor: Unfortunately, pharmaceutical companies and other profit-driven institutions provide nearly all of the funding for the chemistry department's research. Moreover, unless we can secure more funding for basic science research, it is highly unlikely that any significant advances in basic research will come out of the department. Thus, without increased funding from sources other than profit-driven institutions, the chemistry department is unlikely to gain the prestige that only achievements in basic science research confer. Which one of the following is an assumption on which the professor's argument relies? (practice test 10) A) If the chemistry department secures more funding for basic science research its members will make significant advances in basic science B) If the chemistry department's prestige increases substantially, then it is highly likely that the department's funding form other sources other than profit-driven institutions will subsequently increase C) Members of the chemistry department are unlikely to make significant advances in basic science research if the department does not forego the funding it currently receives from profit-driven institutions D) The chemistry department's funding for basic science research is not likely to increase if its funding from sources other than profit-driven institutions does not increase E) The profit-driven institutions that currently provide almost all of the chemistry department's funding are not likely to benefit from basic scientific research

D) The chemistry department's funding for basic science research is not likely to increase if its funding from sources other than profit-driven institutions does not increase

Sociologist: Television, telephones, and other electronic media encourage imprecise, uncritical thinking. Yet critical thinking is the only adequate protection against political demagogues, who seek to exploit people by presenting emotionally loaded language as an objective description of reality. If the sociologist's statements are true, then each of the following statements could be true EXCEPT: (practice test 7) A) There are no political demagogues in some highly technological societies. B) Political demagogues are not the only ones who seek to exploit people by presenting emotionally loaded language as an objective description of reality C) Highly emotional people are more easily exploited than less emotional people D) The mere presence of an orderly system of government in a society provides adequate protection against political demagogues. E) The mere presence of electronic communications technology in a society provides adequate protection against the erosion of media freedoms

D) The mere presence of an orderly system of government in a society provides adequate protection against political demagogues. This can't be true because the stimulus states that the ONLY adequate protection against political demagogues is critical thinking

Advertisement: The dental profession knows that brushing with Blizzard toothpaste is the best way to fight cavities. We surveyed five dentists, and each agreed that the tartar control formula found in Blizzard is the most effective cavity-fighting formula available in a toothpaste. The flawed reasoning in which one of the following is most similar to the flawed reasoning in the advertisement? (practice test 10) A) The nation's voters know that Gomez is the candidate whose policies would be best for the nation. Of 10 voters polled, each said that Gomez would be a very popular leader. B) Some of the nation's voters believe that Gomez is the candidate who would be best for the nation. Of the ten voters we surveyed, each agreed that the policies Gomez is committed to would be the best policies for the nation to adopt. C) The nation's voters generally believe that Gomez is the candidate who would be best for the nation. We polled thousands of voters in the nation, and they agreed that the policies Gomez is committed to would help the nation more than those supported by any of the other candidates D) The nation's voters know that electing Gomez would be the best way to help the nation. The ten voters we polled all agreed that the policies Gomez is committed to would help the nation more than any other policies. E) We know that electing Gomez would be the best course for the nation to follow because, of ten voters we surveyed, each agreed that electing Gomez would help the nation

D) The nation's voters know that electing Gomez would be the best way to help the nation. The ten voters we polled all agreed that the policies Gomez is committed to would help the nation more than any other policies.

Linguist: The Sapir-Whorf hypothesis states that a society's world view is influenced by the language or languages its members speak. But this hypothesis does not have the verifiability of hypotheses of physical science, since it is not clear that the hypothesis could be tested. If the linguist's statements are accurate, which one of the following is most supported by them? (Practice test 5) A) The Sapir-Whorf hypothesis is probably false. B) Only the hypotheses of physical science are verifiable C) Only verifiable hypotheses should be seriously considered D) We do not know whether the Sapir-Whorf hypothesis is true or false E) Only the hypotheses of physical science should be taken seriously

D) We do not know whether the Sapir-Whorf hypothesis is true or false We are told that the Sapir-Whorf hypothesis can't be tested and thus doesn't have the verifiability of physical science hypotheses C: Too extreme

Psychology researchers observed that parents feel emotion while singing to their infants. The researchers hypothesized that this emotion noticeably affects the sound of the singing. To test this hypothesis the parents were recorded while singing to their infants and while singing with no infant present. They were instructed to make the two renditions as similar as possible. These recordings were then played for psychologists who had not observed the recordings being made. For 80 percent of the recordings, these psychologists were able to correctly identify, by listening alone, which recordings were of parents singing to their children. The researchers concluded that their hypothesis was correct. Which one of the following, if true would most strengthen the researchers' reasoning? (practice test 7) A) A separate study by the same researchers found that parents feel more emotion when singing to their own children than when singing to other children. B) Some, but not all, of the parents in the study realized that their song renditions were being recorded C) Parents displayed little emotion when singing with no child or adult present D) When a person feels emotion, that emotion provokes involuntary physiological responses that affect the vocal cords and lungs. E) Most of the parents who participated in the study believed that the emotion they felt while singing to their infants affected their singing

D) When a person feels emotion, that emotion provokes involuntary physiological responses that affect the vocal cords and lungs. Increases the likelihood that the study supports the researcher's hypothesis. Although the results indicate that parents sang DIFFERENTLY when their infant was present, the results don't indicate that emotion CAUSED that difference A: Irrelevant to the given reasoning.

People may praise the talent of a painter capable of realistically portraying a scene and dismiss as artistically worthless the efforts of abstract expressionists, but obviously an exact replica of the scene depicted is not the only thing people appreciate in a painting, for otherwise photography would have entirely displaced painting as an art form. The argument proceeds by (Practice test 3) A) using a claim about what most people appreciate to support an aesthetic principle B) appealing to an aesthetic principle to defend the tastes that people have C) explaining a historical fact in terms of the artistic preferences of people D) appealing to a historical fact to support a claim about people's artistic preferences E) considering historical context to defend the artistic preferences of people

D) appealing to a historical fact to support a claim about people's artistic preferences The historical fact is that photography has NOT entirely displaced painting as an art form A: this choice is too extreme in its focus on what "most people" appreciate

A study found that when rating the educational value of specific children's television shows parents tend to base their judgments primarily on how much they themselves enjoyed the shows, and rarely took into account the views of educational psychologists as to the shows' educational value. Accordingly, if the psychologists' views are sound, parents have little reason to trust their own ratings of the educational value of children's television shows. The argument is most vulnerable to criticism on the grounds that it (Practice test 2) A) relies on a sample that is likely to be unrepresentative of the population with which the conclusion is concerned. B) takes for granted that parents do not enjoy the same sort of children's TV shows that children themselves enjoy C) takes for granted that the educational value of a TV show should be the only consideration for a parent trying to decide whether a child should watch the show D) fails to rule out the possibility that parents' ratings of the shows based on their own enjoyment coincide closely with the educational psychologists' views of the shows' educational values E) takes for granted that educational psychologists are the only people who can judge the educational value of children's TV shows with a high degree of accuracy

D) fails to rule out the possibility that parents' ratings of the shows based on their own enjoyment coincide closely with the educational psychologists' views of the shows' educational values If this is the case, parents may be justified in trusting their own ratings. This introduces a possibility that would obviate the warning in the author's conclusion. A: No reason to suspect that the parents sampled in the study are unrepresentative of the parent population B: Whether parents share their children's taste in TV shows is irrelevant to the conclusion C: Author never says that educational value is the ONLY criterion for choosing one show over another E: Author assumes that educational psychologists can judge the educational value of TV shows, and we have no reason to believe that psychologists are the ONLY people capable of doing so

Scientist: Given the human tendency to explore and colonize new areas, some people believe that the galaxy will eventually be colonized by trillions of humans. If so, the vast majority of humans ever to live would be alive during this period of colonization. Since all of us are humans and we have no reason to think we are unrepresentative, the odds are overwhelming that we would be alive during this period, too. But, because we are not alive during this period, the odds are slim that such colonization will ever happen. The scientist's argument proceeds by (practice test 2) A) reasoning that because an event has not occurred, that event has a low probability of occurring B) drawing a conclusion that implicitly contradicts one of the premises that the argument accepts C) taking for granted that dependable predictions about the future cannot ever be made simply on the basis of present facts D) inferring that since an event that is taken to be likely on a given hypothesis has not occurred, the hypothesis is probably false E) making a prediction far into the future based on established human tendencies

D) inferring that since an event that is taken to be likely on a given hypothesis has not occurred, the hypothesis is probably false Hypothesis: if galaxy will eventually be colonized, we'd likely be alive at the time. Because that hasn't occurred, the hypothesis is probably incorrect A: Rationale behind conclusion is different from the one described above B: Conclusion does not contradict the scientist's premises C: Scientist does not assume that dependable predictions can NEVER be made on the basis of present facts. too extreme E: Misstates the rationale. Not based on human tendencies

Expert: A group of researchers claims to have shown that for an antenna to work equally well at all frequencies, it must be symmetrical in shape and have what is known as a fractal structure. Yet the new antenna developed by these researchers, which satisfies both of these criteria, in fact works better at frequencies below 250 megahertz than at frequencies above 250 megahertz. Hence, their claim is incorrect. The reasoning in the expert's argument is flawed because the argument (Practice test 5) A) fails to provide a definition of the technical term "fractal" B) contradicts itself by denying in its conclusion the claim of scientific authorities that it relies on in its premises C) concludes that a claim is false merely on the grounds that there is insufficient evidence that it is true D) interprets an assertion that certain conditions are necessary as asserting that those conditions are sufficient E) takes for granted that there are only two possible alternatives, either below or above 250 megahertz

D) interprets an assertion that certain conditions are necessary as asserting that those conditions are sufficient E: difference between frequencies isn't critical to the argument at all

University president: Research institutions have an obligation to promote research in any field of theoretical investigation if that research shows some promise of yielding insights into the causes of practical problems that affect people's quality of life. The principle stated by the university president, if valid, most helps to justify which of the following actions? (practice test 1) A) A university denies a grant application from a faculty member for work on a solution to a famous mathematical puzzle that has no relation to practical concerns. B) A government agency funds a research project in astrophysics designed to determine whether there are theoretical limits on the magnitude of planets in distant solar systems. C) A university funds a research position in the physics department that involves no teaching but has the responsibility for managing all the grant applications by members of the physics faculty. D) A foundation decides not to fund a research proposal in applied math that sought to model certain poorly understood aspects of economic behavior. E) A research institute funds an investigation into the mathematical properties of folded structures that is likely to aid in the understanding of the structure of proteins that cause disease.

E) A research institute funds an investigation into the mathematical properties of folded structures that is likely to aid in the understanding of the structure of proteins that cause disease. A and D: Stimulus tells us when research institutions have an obligation to PROMOTE research, not DENY research. B and C: Stimulus focuses on causes of practical problems that affect people's lives, and this is not addressed in the choice.

The chairperson of Acme Corporation has decided to move the company from its current location in Milltown to Ocean View. Most Acme employees cannot afford housing within a 30-minute commute of Ocean View. So once the company has moved, most Acme employees will have a commute of more than 30 minutes. This argument requires assuming which of the following? (practice test 2) A) All Acme employees can afford housing within a 30-minute commute of Milltown B) The chairperson of Acme has good financial reasons for wanting to move the company to Ocean View C) None of Acme's employees except the chairperson are in favor of moving the company to Ocean View D) Currently, most Acme employees have a commute of less than 30 minutes E) Acme's move to Ocean View will not be accompanied by a significant pay raise for Acme employees

E) Acme's move to Ocean View will not be accompanied by a significant pay raise for Acme employees If the opposite of the choice weakens the conclusion, we can confirm it's the answer; if employees did get a significant pay raise, perhaps they could afford housing within a 30-minute commute of Ocean View A: Author makes no assumptions about how many Acme employees can afford housing within 30-minutes of Milltown B: Claim is unnecessary C: Irrelevant; and no assumptions are made in this respect D: Argument makes no assumptions about Acme employees' CURRENT commute time

Editor: Most of the books of fiction we have published were submitted by literary agents for writers they represented; the rest were received directly from fiction writers from whom we requested submissions. No nonfiction manuscript has been given serious attention, let alone been published, unless it was from a renowned figure or we had requested the manuscript after careful review of the writer's book proposal. Which one of the following can be properly inferred from the editor's statements? (practice test 3) A) Most unrequested manuscripts that the publishing house receives are not given serious attention. B) Most of the books that the publishing house publishes that are not by renowned authors are books of fiction C) If a manuscript has received careful attention at the publishing house, then it is either a work of fiction or the work of a renowned figure. D) The publishing house is less likely to give careful consideration to a manuscript that was submitted directly by a writer than one that was submitted by a writer's literary agent E) Any unrequested manuscripts not submitted by literary agents that the publishing house has published were written by renowned figures

E) Any unrequested manuscripts not submitted by literary agents that the publishing house has published were written by renowned figures According to the stimulus, a published fiction manuscript must either be requested or submitted by an agent; any published nonfiction manuscript must be by a renowned figure or requested. So if it was not requested or by an agent, it must be by a renowned figure A: We are not told what proportion of requested or unrequested manuscripts are taken seriously

For several centuries there have been hairless dogs in western Mexico and in coastal Peru. It is very unlikely that a trait as rare as hairlessness emerged on two separate occasions. Since the dogs have never existed in the wild, and the vast mountainous jungle separating these two regions would have made overland travel between them extremely difficult centuries ago, the dogs must have been transported from one of these regions to the other by boat, probably during trading expeditions. Which one of the following is an assumption that the argument requires? (practice test 6) A) Hairless dogs have never been found anywhere except in the regions of western Mexico and coastal Peru. B) Most of the trade goods that came into western Mexico centuries ago were transported by boat. C) Centuries ago, no one would have traveled between western Mexico and coastal Peru by boat except for the purposes of carrying out a trading expedition. D) If hairless dogs were at one time transported between western Mexico and coastal Peru by boat, they were traded in exchange for other goods. E) Centuries ago, it was easier to travel by boat between western Mexico and coastal Peru than to travel by an overland route.

E) Centuries ago, it was easier to travel by boat between western Mexico and coastal Peru than to travel by an overland route. This answer is best arrived upon by approaching it with a clear understanding of the argument and then using the negation test; if negating the choice causes the argument to fall apart, we found the answer. A: Premises all relate to the difficulties of getting between the two areas centuries ago; this doesn't relate. B: It doesn't have to be true that MOST of the trade goods coming into western Mexico at the time came by boat as long as the ones from Peru did. C: Too strong; not required that trading be the ONLY reason that people travel. D: Reason for transport isn't necessary in the argument.

Insurers and doctors are well aware that the incidence of lower-back injuries among office workers who spend long hours sitting is higher than that among people who regularly do physical work of a type known to place heavy stresses on the lower back. This shows that office equipment and furniture are not properly designed to promote workers' health. Which one of the following, if true, most undermines the reasoning above? (practice test 11) A) When they are at home, laborers and office workers tend to spend similar amounts of time sitting B) Insurance companies tend to dislike selling policies to companies whose workers often claim to have back pain C) People who regularly do physical work of a type known to place heavy stress on the lower back are encouraged to use techniques that reduce the degree of stress involved D) Most of the lower-back injuries that office workers suffer occur while they are on the job E) Consistent physical exercise is one of the most effective ways to prevent or recover from lower back injuries

E) Consistent physical exercise is one of the most effective ways to prevent or recover from lower back injuries

If a corporation obtains funds fraudulently, then the penalty should take into account the corporation's use of those funds during the time it held them. In such cases, the penalty should completely offset any profit the corporation made in using the funds. Which of the following conforms most closely to the principle illustrated above? (Practice Test 6) A) If a driver causes an accident because the automobile being driven was not properly maintained, that driver should be required from then on to regularly demonstrate that his or her automobile is being properly maintained. B) If a factory is found to have been recklessly violating pollution laws, that factory should be required to make the expenditures necessary to bring it into compliance with those laws to the satisfaction of regulators. C) If someone is sentenced to perform community service, the court has a responsibility to ensure that the community at large rather than a private group benefits from that service. D) If an athlete is found to have used banned performance-enhancing substances, that athlete should be prohibited from participating in all future athletic competitions. E) If a convicted criminal writes a memoir describing the details of that criminal's crime, any proceeds of the book should be donated to a charity chosen by a third party.

E) If a convicted criminal writes a memoir describing the details of that criminal's crime, any proceeds of the book should be donated to a charity chosen by a third party. Principle: One shouldn't be able to profit from one's crime, and the subsequent penalty should account for that. A:No ill-gained profit that needs to be addressed. B: No ill-gained profit that needs to be addressed. C: No ill-gained profit that needs to be addressed. D: Athlete likely profited by using illegal substances, but the penalty doesn't offset that profit.

Unusually large and intense forest fires swept the tropics in 1997. The tropics were quite susceptible to fire at that time because of the widespread drought caused by an unusually strong El Niño, an occasional global weather phenomenon. Many scientists believe the strength of the El Niño was enhanced by the global warming caused by air pollution. Which one of the following can be properly inferred from the information above? (Practice test 4) A) Air pollution was largely responsible for the size and intensity of the forest fires that swept the tropics in 1997. B) If the El Nino in 1997 had not been unusually strong, few if any large and intense forest fires would have swept the tropics in that year C) Forest fires in the tropics are generally larger and more intense than usual during a strong El Nino D) At least some scientists believe that air pollution was responsible for the size and intensity of the forest fires that swept the tropics in 1997 E) If air pollution enhanced the strength of the El Nino in 1997, then it also contributed to the widespread drought in that year

E) If air pollution enhanced the strength of the El Nino in 1997, then it also contributed to the widespread drought in that year We cannot be sure that the scientists' belief about what caused the strength of the El Nino is correct, but IF it is, then it logically follows that air pollution contributed to the drought D: Just because we hold a certain belief doesn't mean that we also believe in its logical implications. The stimulus stated that the scientists believe that air pollution caused global warming, and that's ALL they believe

Consultant: The dramatic improvements in productivity achieved during the Industrial Revolution resulted in large part from standardization of processes and procedures coupled with centralization of planning and decision making. Yet, in recent years, many already productive companies have further improved their productivity by giving individual employees greater influence in decision making and in how they do their work. Which one of the following, if true, most helps to resolve the apparent paradox in the consultant's statements? (practice test 3) A) Most companies still try to improve productivity mainly through greater standardization and centralization of decision making B) Increased productivity is not the only benefit of giving individual employees greater control over their work; job satisfaction increases as well C) Most of the increases in industrial productivity that have occurred in recent years have been due to the introduction of advanced technology like industrial robots D) The innovations of the industrial Revolution are only now being applied in those companies in which individual employees have traditionally been entirely in control of how they do their work E) Increases in productivity in highly productive companies depend on management's broad application of innovative ideas solicited from individual employees about their work

E) Increases in productivity in highly productive companies depend on management's broad application of innovative ideas solicited from individual employees about their work This indicates that ideas solicited from individual employees are crucial to increasing productivity, which explains the discrepancy B: Knowing an additional benefit of autonomy doesn't resolve the paradox

Advertisement: In a recent survey, a sample representative of all new Popelka Auto Insurance policyholders reported savings of $250 a year, on average, as a result of switching their auto insurance coverage to Popelka. Thus, most people who hold auto insurance policies with other companies could save hundreds of dollars by switching to Popelka. The argument in the advertisement is most vulnerable to criticism on which one of the following grounds? (practice test 3) A) It overlooks the possibility that at least some of the new Popelka Auto Insurance policyholders surveyed reported that they saved little or no money when they switched their auto insurance coverage to Popelka. B) It takes for granted that the new Popelka Auto Insurance policyholders pay no less for their auto insurance, on average, than do people who have help Popelka Auto Insurance policies for a longer period of time. C) It fails to address adequately the possibility that switching to another insurance company would enable many auto insurance policyholders to save even more money than they would save by switching to Popelka D) It takes for granted that few if any of the Popelka Auto Insurance policyholders surveyed underestimated how much they saved when they switched their auto insurance coverage to Popelka E) It fails to address adequately the possibility that people capable of saving hundreds of dollars by switching their auto insurance coverage to Popelka are disproportionately represented among the new Popelka auto insurance policyholders

E) It fails to address adequately the possibility that people capable of saving hundreds of dollars by switching their auto insurance coverage to Popelka are disproportionately represented among the new Popelka auto insurance policyholders Argument concludes that "most people" could save; but this is based on a representative sample of NEW POPELKA POLICYHOLDERS who may be people whose previous insurance was overpriced

Linguist: One group of art critics claims that postimpressionist paintings are not really art and so should be neither studied nor displayed. Another group of critics disagrees, insisting that these paintings are works of art. But since the second group grants that there are paintings that are not works of art and should therefore be ignored in the manner suggested by the first group, their disagreement is not over the meaning of the word "art." The claim that there are paintings that are not works of art plays which one of the following roles in the linguist's argument? (practice test 10) A) It is a contention that the argument purports to show is the main point of disagreement between the two groups of critics mentioned B) It is cited as a commonly accepted reason for accepting a hypothesis for which the argument offers independent evidence C) It is a claim whose acceptance by critics who differ on the other issues is cited by the argument as evidence of its truth D) It is a claim about the nature of art that according to the argument accounts for disputes that only appear to concern the aesthetic merits of certain types of paintings E) It is a claim whose acceptance by both of the two disputing parties is cited as evidence for a conclusion the argument draws about the disagreement.

E) It is a claim whose acceptance by both of the two disputing parties is cited as evidence for a conclusion the argument draws about the disagreement.

Botanist: In an experiment, scientists raised domesticated radishes in a field with wild radishes, which are considered weeds. Within several generations, the wild radishes began to show the same flower color as the domesticated ones. This suggests that resistance to pesticides, which is often a genetically engineered trait, would also be passed from domesticated crop plants to their relatives that are considered weeds. Which one of the following, if true, most strengthens the botanist's argument? (Practice test 4) A) It is much easier in principle for genetic traits to be passed from wild plants to their domesticated relatives than it is for such traits to be passed from the domesticated plant to the wild relative B) When the ratio of domesticated radishes to wild radishes in the field increased, the speed with which the flower color passed to the wild radishes also increased. C) Radishes are not representative of crop plants in general with respect to the ease with which various traits are passed among members of closely related species D) The flower color of the domesticated radishes had not been introduced into them via genetic engineering E) It is more difficult for flower color to be transferred between domesticated and wild radishes than it is for almost any other trait to be passed between any two similarly related plant species

E) It is more difficult for flower color to be transferred between domesticated and wild radishes than it is for almost any other trait to be passed between any two similarly related plant species Strengthens the presumed analogy between flower color and pesticide resistance. If flower color is one of the most difficult traits for radishes to transfer, then it logically follows that other traits would be even easier to transfer B: Doesn't strengthen the connection between conclusion and evidence because the conclusion is about resistance to pesticides, not flower color

Editorial: In order to encourage personal responsibility in adults, society should not restrict the performance of any of the actions of adults or interfere with the likely results except to prevent negative effects on others. Which one of the following expresses a view that is inconsistent with the principle stated in the editorial? (Practice test 4) A) We should not prevent the students from wasting the classroom time set aside for homework. But this does not mean that they may spend the time any way that they wish. Activities disruptive to others should not be tolerated. B) The scientist who invented this technology is not the only one who should be allowed to profit from it. After all, there is no evidence that allowing others to profit from this technology will reduce the scientist's own profits. C) Even though public smoking may lead to indirect harm to others, it should not be banned. There are several other ways to eliminate harm that do not restrict the conduct of smokers and are hence preferable to a complete ban on public smoking. D) Highway speed limits are a justified restriction of freedom. For drivers who speed do not risk only their own lives; such drivers often injure or kill other people. Moreover, speed limits have been shown to significantly reduce highway accident and fatality rates. E) It is not enough that consumable products containing harmful substances have warning labels. Many adults simply ignore such warnings and continue to consume these substances in spite of the harm it may cause them. This is why consuming such substances should be illegal.

E) It is not enough that consumable products containing harmful substances have warning labels. Many adults simply ignore such warnings and continue to consume these substances in spite of the harm it may cause them. This is why consuming such substances should be illegal. This view advocates legislation that would prohibit adults from harming THEMSELVES, contradictory to the stimulus view. B: the stimulus does not address conditions under which profits should be shared so this view cannot contradict the principle in the stimulus

The goblin fern, which requires a thick layer of leaf litter on the forest floor, is disappearing from North American forests. In spots where it has recently vanished, the leaf litter is unusually thin and, unlike those places where this fern still thrives, is teeming with the European earthworm Lumbricus rubellus, which eats leaf litter. L. rubellus is thus probably responsible for the fern's disappearance. Which one of the following is an assumption on which the argument depends? (Practice test 4) A) Wherever there is a thick layer of leaf litter in North American forests, goblin ferns can be found. B) None of the earthworms that are native to North America eat leaf litter C) Dead leaves from goblin ferns make up the greater part of the layer of leaf litter on the forest floors where the goblin fern has recently vanished D) There are no spots in the forests of North America where both goblin ferns and earthworms of the species L. rubellus can be found. E) L. rubellus does not favor habitats where the leaf litter layer is considerably thinner than what is required by goblin ferns.

E) L. rubellus does not favor habitats where the leaf litter layer is considerably thinner than what is required by goblin ferns. The line of reasoning in the stimulus assumes that the earthworms are responsible for the thinning of leaf litter. If the opposite of this choice were true, the earthworms wouldn't be causing anything because they would be attracted to the kind of habitat that the fern can't thrive in D: The argument does not require that goblin ferns and earthworms NEVER coexist; in fact, we can assume that wherever goblin fern recently vanished, there was a period of time when both species were found there

Most auto mechanics have extensive experience. Furthermore, most mechanics with extensive experience understand electronic circuits. Thus, most auto mechanics understand electronic circuits. The pattern of flawed reasoning in which one of the following arguments is most similar to that in the argument above? (practice test 9) A) During times of the year when automobile traffic increases, gas prices also increase. Increases in gas prices lead to increases in consumer complaints. Thus, increased automobile traffic causes increased consumer complaints B) The most common species of birds in this region are migratory. Moreover, most migratory birds have left this region by the end of November, Hence, few birds remain in this region during winter C) It is not surprising that most speeding tickets in this region are issued to drivers of sports cars. After all, most drivers who are not interested in driving fast do not buy sports cars D) Most nature photographers find portrait photography boring. Moreover, most portrait photographers especially enjoy photographing dignitaries. Thus, most nature photographers find photographing dignitaries especially boring E) Most snow-removal companies run lawn-care services during the summer. Also, most companies that run lawn-care services during the summer hire additional workers in the summer. Thus, most snow-removal companies hire additional workers in the summer

E) Most snow-removal companies run lawn-care services during the summer. Also, most companies that run lawn-care services during the summer hire additional workers in the summer. Thus, most snow-removal companies hire additional workers in the summer Most X are Y and most Y are Z. Therefore, most X are Z

Well-intentioned people sometimes attempt to resolve the marital problems of their friends. But these attempts are usually ineffectual and thereby foster resentment among all parties. Thus, even well-intentioned attempts to resolve the marital problems of friends are usually unjustified. Which of the following principles, if valid, most strongly supports the reasoning above? (Practice test 6) A) One should get involved in other people's problems only with the intention of producing the best overall consequences. B) Interpersonal relations should be conducted in accordance with doing whatever is right, regardless of the consequences. C) Good intentions are the only legitimate grounds on which to attempt to resolve the marital problems of friends. D) The intentions of an action are irrelevant to whether or not that action is justified. E) No actions based on good intentions are justified unless they result in success.

E) No actions based on good intentions are justified unless they result in success. A: Neither strengthens nor weakens the passage's argument. B: Weakens the passage's reasoning by disregarding the importance of consequences, which the passage prioritizes as support for recommending against an action. C: We are looking for a principle that helps confirm the conclusion that well-intentioned attempts usually aren't justified, which this doesn't provide. D: This choice doesn't help the support bridge to the given conclusion; the suggestion this choice focuses on is not the core of the argument. This choice doesn't connect an act's being ineffectual and fostering resentment to that act being unjustified.

Professor: The number of new university students who enter as chemistry majors has not changed in the last ten years, and job prospects for graduates with chemistry degrees are better than ever. Despite this, there has been a significant decline over the past decade in the number of people earning chemistry degrees. Which of the following, if true, most helps to explain the decline? (Practice test 6) A) Many students enter universities without the academic background necessary for majoring in chemistry. B) There has been a significant decline in the number of undergraduate degrees earned in the natural sciences as a whole. C) Many students are very unsure of their choice when they pick a major upon entering universities. D) Job prospects for graduates with chemistry degrees are no better than prospects for graduates with certain other science degrees. E) Over the years, first-year chemistry has come to be taught in a more routinely methodical fashion, which dampens the intellectual appeal.

E) Over the years, first-year chemistry has come to be taught in a more routinely methodical fashion, which dampens its intellectual appeal. If fewer people are interested in pursuing chem after experiencing first-year chem classes, many students may change their majors. A: We would need to know what has changed to cause a decline even though job prospects for chem degrees are strong. B: Doesn't explain WHY number of degrees are declining. C: doesn't help to explain the decline over the past decade in chem degrees. would have to know what has changed. D: doesn't help to explain the decline in numbers of chem degrees; if job prospects are better than ever, why are fewer people earning chem degrees?

Sociologist: Romantics who claim that people are not born evil but may be made evil by the imperfect institutions that they form cannot be right, for they misunderstand the causal relationship between people and their institutions. After all, institutions are merely collections of people. Which one of the following principles, if valid, would most help to justify the sociologist's argument? (LSAT Max Practice Test) A) People acting together in institutions can do more good or evil than can people acting individually. B) Institutions formed by people are inevitably imperfect. C) People should not be overly optimistic in their view of individual human beings. D) A society's institutions are the surest gauge of that society's values. E) The whole does not determine the properties of the things that compose it.

E) The whole does not determine the properties of the things that compose it. Romantics believe that the whole (the institution) impacts the parts (the individuals), whereas the sociologist believes this is wrong and the whole can't determine the characteristics of the parts (in fact, the causality is the other way around)

Richard: Because it fails to meet the fundamental requirement of art—that it represent—abstract art will eventually be seen as an aberration. Jung-Su: Although artists, like musicians, may reject literal representation, makers of abstract art choose to represent the purely formal features of objects, which are discovered only when everyday perspectives are rejected. Thus, whatever others might come to say, abstract art is part of the artistic mainstream. Richard and Jung-Su disagree over whether (practice test 6) A) makers of abstract art reject literal representation B) the fundamental requirement of art is that it represent C) musicians may reject literal representation D) abstract art will be seen as an aberration E) abstract art is representational

E) abstract art is representational We can infer that Jung-Su believes that abstract art is representational based on the statement that "...choose to REPRESENT the purely formal features...." A: speakers seem to agree on this idea B: Jung-Su doesn't affirm or deny this, and instead treats the question of whether abstract art meets that requirement C: this isn't mentioned as a point of disagreement since Richard has no comment on musicians. D: Jung-Su's rebuttal doesn't concern whether abstract art will be seen as an aberration, but rather what abstract art really is. Phrase "whatever others might come to say" also implies that Jung-Su isn't disagreeing that some people may view it as an aberration.

Mark: The decongestant drug Zokaz was discontinued by its manufacturer because long-term studies revealed that it increased the risk of heart attack. Qualzan, another decongestant, works by essentially the same physiological mechanism as Zokaz. So Qualzan probably also increases the risk of heart attack. Kathy: The decongestive effects of the two drugs do stem from the same physiological mechanism. But since they are different chemically, the two drugs probably have different side effects. Which one of the following is a technique of reasoning used in Kathy's response to Mark? (practice test 3) A) using a product's overall record of safety as evidence that the product is not linked to a particular health program B) attempting to discredit an argument by comparing it to another obviously flawed argument that is logically parallel C) arguing against a conclusion by raising questions about the validity of scientific studies cited in support of that conclusion D) attempting to undermine an argument by showing that it is incompatible with a fundamental principle of medicine E) challenging an argument from analogy by focusing on a dissimilarity between the things being compared

E) challenging an argument from analogy by focusing on a dissimilarity between the things being compared Kathy implies that while the two drugs ARE similar in one way, they are different in another way and therefore Mark's analogy may not be sound. D: Kathy doesn't cite any fundamental principles of medicine or take issue with principles underlying Mark's argument

When a group is unable to reach a consensus, group members are often accused of being stubborn, bull-headed, or unyielding. Such epithets often seem abusive, are difficult to prove, and rarely help the group reach a resolution. Those who wish to make such an accusation stick, however, should choose "unyielding," because one can always appeal to the fact that the accused has not yielded; obviously if one acknowledges that a person has not yielded, then one cannot deny that the person is unyielding, at least on this issue. Which of the following most accurately describes the argumentative technique employed above? (practice test 1) A) rejecting a tactic on the grounds that it constitutes an attack on the character of a person and has no substance in fact. B) rejecting a tactic on the grounds that the tactic makes it virtually impossible for the group to reach a consensus on the issue in question C) conditionally advocating a tactic on the grounds that it results in an accusation that is less offensive than the alternatives. D) conditionally advocating a tactic on the grounds that it results in an argument that would help the group to reach a consensus on the issue in question. E) conditionally advocating a tactic on the grounds that it results in an argument for which one could not consistently accept the premise but deny the conclusion.

E) conditionally advocating a tactic on the grounds that it results in an argument for which one could not consistently accept the premise but deny the conclusion. The arguer recommends the specific words because they're easier to prove. A: Arguer doesn't reject a tactic; they instead recommend a specific choice among three options. B: Doesn't reject a tactic; they instead recommend a specific choice among three options C: Arguer doesn't provide these recommendations based on level of offensiveness, but rather because it's easier to prove. D: Arguer doesn't provide these recommendations because they're best suited for the goal of consensus, but because they're easier to prove.

All the apartments on 20th Avenue are in old houses. However, there are twice as many apartments on 20th Avenue as there are old houses. Therefore, most old houses on 20th Avenue contain more than one apartment. The reasoning in the argument above is most vulnerable to criticism on the grounds that the argument (practice test 11) A) overlooks the possibility that some of the buildings on 20th avenue are not old houses B) draws a conclusion that simply restates one of the premises offered in support of the conclusion C) fails to consider the possibility that some buildings on 20th avenue may offer types of rental accommodation other than apartments D) confuses a condition whose presence would be sufficient to ensure the truth of the argument's conclusion with a condition whose presence is required in order for the conclusion to be true E) fails to address the possibility that a significant number of old houses on 20th avenue contain three or more apartments

E) fails to address the possibility that a significant number of old houses on 20th avenue contain three or more apartments

An analysis of the language in social media messages posted via the Internet determined that, on average, the use of words associated with positive moods is common in the morning, decreases gradually to a low point midafternoon, and then increases sharply throughout the evening. This shows that a person's mood typically starts out happy in the morning, declines during the day, and improves in the evening. The reasoning in the argument is most vulnerable to criticism on the grounds that the argument overlooks the possibility that (practice test 11) A) people's overall moods are lowest at the beginning of the workweek and rise later, peaking on the weekend B) many people who post social media messages use neither words associated with positive moods nor words associated with negative moods C) the frequency in the use of words in social media is not necessarily indicative of the frequency of the use of those words in other forms of communication D) the number of social media messages posted in the morning is not significantly different from the number posted in the evening E) most of the social media messages posted in the evening are posted by people who rarely post such messages in the morning

E) most of the social media messages posted in the evening are posted by people who rarely post such messages in the morning

Sigerson argues that the city should adopt ethical guidelines that preclude its politicians from accepting campaign contributions from companies that do business with the city. Sigerson's proposal is dishonest, however, because he has taken contributions from such companies throughout his career in city politics. The reasoning in the argument is most vulnerable to criticism on the grounds that the argument (Practice test 9) A) confuses a sufficient condition for adopting ethical guidelines for politicians with a necessary condition for adopting such guidelines B) rejects a proposal on the grounds that an inadequate argument has been given for it C) fails to adequately address the possibility that other city politicians would resist Sigerson's proposal D) rejects a proposal on the grounds that the person offering it is unfamiliar with the issues it raises E) overlooks the fact that Sigerson's proposal would apply only to the future conduct of city politicians

E) overlooks the fact that Sigerson's proposal would apply only to the future conduct of city politicians The proposal isn't dishonest just because of the way he used to act; they could have realized that their past actions shouldn't be allowable in the future

Plumb-Ace advertises that its plumbers are more qualified than plumbers at any other major plumbing firm in the region because Plumb-Ace plumbers must complete a very difficult certification process. Plumb-Ace plumbers may or may not be more qualified, but clearly the certification process is not very difficult, because nearly everyone who takes the written portion of the certification exam passes it very easily. The reasoning in the argument is flawed in that it (Practice test 4) A) treats something that is necessary to make a certification process very difficult as if it were sufficient by itself to make the process very difficult B) takes for granted that plumbers are not qualified unless they complete some certification process C) overlooks the possibility that plumbers at other firms in the region complete certification processes that are even easier than that completed by Plumb-Ace's plumbers D) infers that a claim is false on the grounds that an inadequate argument has been given for the claim E) presumes that since one part of a whole lacks a certain characteristic, the whole must lack that characteristic as well.

E) presumes that since one part of a whole lacks a certain characteristic, the whole must lack that characteristic as well. Just because one part of it - the written exam - isn't very difficult, the whole process can, of course, still be quite difficult. Be careful not to confuse background information with the actual argument. C: If other certification process are easier, this could only undermine Plumb-Ace's line of reasoning, not the author's

Practice Test 6 Merton: A study showed that people who live on very busy streets have higher rates of heart disease than average. I conclude that this elevated rate of heart disease is caused by air pollution from automobile exhaust. Ortiz: Are you sure? Do we know whether people living on busy streets have other lifestyle factors that are especially conducive to heart disease? Ortiz criticizes Merton's argument by: A) raising a question about the validity of the study that Merton cites. B) contending that Merton needs to take into account other effects of air pollution C) claiming that Merton misunderstands a crucial aspect of the study's findings D) raising a counterexample to the general conclusion that Merton draws E) suggesting that alternative explanations for the study's findings need to be ruled out

E) suggesting that alternative explanations for the study's findings need to be ruled out. Ortiz reminds Merton there could be other factors affecting elevated rates of heart disease. Choice A: Ortiz doesn't question the study, he questions the conclusion that Merton arrives at. Choice B: Ortiz contends that they must take into account alternative causes of high heart disease rates, NOT alternative effects of air pollution. Choice C: The study is not misunderstood, the conclusion that Merton arrives at is faulty. Choice D: Ortiz doesn't raise a counterexample; a counterexample would be something like "Are you sure? Is it possible that people living on busy streets have a poor diet due to the abundance of fast food on their streets?"

Studies have found that human tears contain many of the same hormones that the human body produces in times of emotional stress. Hence, shedding tears removes significant quantities of these hormones from the body. Therefore, crying must have the effect of reducing emotional stress. The reasoning in the argument is most vulnerable to criticism on the grounds that the argument (practice test 2) A) overlooks the possibility that if crying has a tendency to reduce emotional stress, this tendency might arise because of something other than the shedding of tears B) confuses a condition that is required for the production of a given phenomenon with a condition that in itself would be sufficient to cause the production of that phenomenon C) fails to adequately address the possibility that, even if one phenomenon causally contributes to a second phenomenon, the second phenomenon may causally influence the first as well D) fails to adequately distinguish between two distinct factors that are jointly responsible for causing a given phenomenon E) takes for granted that because certain substances are present whenever a condition occurs, those substances are a cause of that condition

E) takes for granted that because certain substances are present whenever a condition occurs, those substances are a cause of that condition Author assumes that, since certain substances are present whenever a condition occurs, those are the cause of that condition A: Irrelevant B: This choice describes an error in conditional reasoning, although the major flaw in the argument is causal C: Author must consider whether hormones cause stress in the first place, or if there is a feedback loop D:The greatest flaw in the argument is that correlation implies causation

Editorial: Painting involves a sequential application of layers, each of which adheres satisfactorily only if the underlying layer has been properly applied. Education is, in this respect, like the craft of painting. Since the most important steps in painting are preparation of the surface to be painted and application of the primer coat, it makes sense to suppose that __________. Which one of the following most logically completes the editorial's argument? (practice test 2) A) in the educator's initial contact with a student, the educator should be as undemanding as possible B) students who have a secure grasp of the fundamentals of a subject are likely to make progress in that subject C) educators who are not achieving the goals they intended should revise their teaching methods D) teaching new students is rewarding but much more difficult than teaching more advanced students E) the success of a student's overall educational experience depends above all upon that student's initial educational experience

E) the success of a student's overall educational experience depends above all upon that student's initial educational experience Proper foundation is ESSENTIAL for painting, so it makes sense that it is also ESSENTIAL to education A: No reason given to believe that teachers should be undemanding in their initial contact with a student B: Language is not strong enough ("Likely to make progress" vs. "Essential") C: the need for educators to revise teaching methods is not inferred from information provided D: Irrelevant

Philosopher: Nations are not literally persons; they have no thoughts or feelings, and, literally speaking, they perform no actions. Thus they have no moral rights or responsibilities. But no nation can survive unless many of its citizens attribute such rights and responsibilities to it, for nothing else could prompt people to make the sacrifices national citizenship demands. Obviously, then, a nation _________. Which one of the following most logically completes the philosopher's argument? (LSAT Max Practice Test) A) cannot continue to exist unless something other than the false belief that the nation has moral rights motivates its citizens to make sacrifices B) cannot survive unless many of its citizens have some beliefs that are literally false C) can never be a target of moral praise or blame D) is not worth the sacrifices that its citizens make on its behalf E) should always be thought of in metaphorical rather than literal terms

cannot survive unless many of its citizens have some beliefs that are literally false Argument Completion questions usually present two ideas then bring them together in the conclusion; here, the two ideas are that 1) nations aren't actually people, but 2) their survival depends on citizens believing that they are. This is leading to a conclusion about a nation requiring people to believe something that isn't true in order to survive

Columnist: It may soon be possible for an economy to function without paper money. Instead, the government would electronically record all transactions as they take place. However, while this may be technologically feasible it would never be willingly accepted by a society, for it gives the government too much power. People are rightly distrustful of governments with too much power. Which one of the following most accurately expresses the overall conclusion of the columnist's argument? (practice test 1) A) A society would never willingly accept a system in which, in lieu of paper money, the government keeps track of every transaction electronically. B) It is reasonable for people to distrust a government that has too much power. C) New technology may soon make it possible for an economy to operate without paper money. D) People are right to be unwilling to give the government the power it would need to operate an economy without paper money. E) Even though it may be technologically feasible, no government will be able to operate an economy without the use of paper money.

A) A society would never willingly accept a system in which, in lieu of paper money, the government keeps track of every transaction electronically. Matches the statement made in the passage beginning with "However..." B: Although it matches the last sentence of the stimulus, it is not the overall conclusion. C: This statement matches the first sentence, but that's not the overall conclusion. D: Matches the last sentence in the stimulus, which isn't the main conclusion. E: This claim isn't present anywhere in the stimulus.

Economics professor: Marty's Pizza and Checkers Pizza are the two major pizza parlors in our town. Marty's sold coupon books including coupons good for one large plain pizza at any local pizza parlor, at Marty's expense. But Checkers refused to accept these coupons, even though they were redeemed by all other local pizza parlors. Accepting them would have cost Checkers nothing and would have satisfied those of its potential customers who had purchased the coupon books. This shows that Checkers's motive in refusing to accept the coupons was simply to hurt Marty's Pizza. Which one of the following, if assumed, enables the economics professor's conclusion to be properly drawn? (practice test 9) A) Any company that refuses to accept coupons issued by a competitor when doing so would satisfy some of the company's potential customers is motivated solely by the desire to hurt their competitor B) Any company that wishes to hurt a competitor by refusing to accept coupons issued by that competitor will refuse to accept them even when accepting them would cost nothing and would satisfy its potential customers C) At least one company has refused to accept coupons issued by its major local competitor simply in order to hurt that competitor, even though those coupons were accepted by all other local competitors D) Any company that accepts its major competitor's coupons helps its competitor by doing so, even if it also satisfies its own actual potential customers E) If accepting coupons issued by a competitor would not enable a company to satisfy its actual or potential customers, then that company's refusal to accept the coupons is motivated by the desire to satisfy customer

A) Any company that refuses to accept coupons issued by a competitor when doing so would satisfy some of the company's potential customers is motivated solely by the desire to hurt their competitor


Conjuntos de estudio relacionados

biology chapter 8 study questions

View Set

Lecture 5: Measures of Association

View Set

Study Guide for Thursday's Literature Test

View Set

Medical Terminology - Chapter 4 - Musculoskeletal System

View Set